Anda di halaman 1dari 50

Teora elemental de los nmeros para olimpadas matemticas c

David A. SANTOS

Revisin del 17 de julio de 2008

ndice general

Prefacio 1. Preliminares 1.1. Buen orden . . . . . . . . . . . . . . . Tarea . . . . . . . . . . . . . . . . . . . . . . 1.2. Induccin . . . . . . . . . . . . . . . . Tarea . . . . . . . . . . . . . . . . . . . . . . 1.3. Principio de las pichoneras de Dirichlet Tarea . . . . . . . . . . . . . . . . . . . . . .

III

. . . . . .

. . . . . .

. . . . . .

. . . . . .

. . . . . .

. . . . . .

. . . . . .

. . . . . .

. . . . . . . . . . . . . .

. . . . . . . . . . . . . .

. . . . . . . . . . . . . .

. . . . . . . . . . . . . .

. . . . . . . . . . . . . .

. . . . . . . . . . . . . .

. . . . . . . . . . . . . .

. . . . . . . . . . . . . .

. . . . . . . . . . . . . .

. . . . . . . . . . . . . .

. . . . . . . . . . . . . .

. . . . . . . . . . . . . .

. . . . . . . . . . . . . .

. . . . . . . . . . . . . .

. . . . . . . . . . . . . .

. . . . . . . . . . . . . .

. . . . . . . . . . . . . .

. . . . . . . . . . . . . .

. . . . . . . . . . . . . .

. . . . . . . . . . . . . .

. . . . . . . . . . . . . .

. . . . . . . . . . . . . .

. . . . . . . . . . . . . .

. . . . . . . . . . . . . .

. . . . . . . . . . . . . .

. . . . . . . . . . . . . .

. . . . . . . . . . . . . .

. . . . . . . . . . . . . .

. . . . . . . . . . . . . .

. . . . . . . . . . . . . .

1 1 2 2 3 3 3 4 4 8 9 10 10 12 13 15 16 16 21 23 29 30 30 32 33 35 35 37 37 38 39 41

2. Divisibilidad y primos 2.1. Divisibilidad y algoritmo de divisin . . . . . . . . . . Tarea . . . . . . . . . . . . . . . . . . . . . . . . . . . . . . 2.2. Mximo comn divisor . . . . . . . . . . . . . . . . . Tarea . . . . . . . . . . . . . . . . . . . . . . . . . . . . . . 2.3. Primos y factorizacin nica . . . . . . . . . . . . . . Tarea . . . . . . . . . . . . . . . . . . . . . . . . . . . . . . 2.4. Algoritmo de Euclides y ecuaciones diofnticas lineales Tarea . . . . . . . . . . . . . . . . . . . . . . . . . . . . . . 3. Funciones aritmticas 3.1. Funciones multiplicativas Tarea . . . . . . . . . . . . . . 3.2. La funcin parte entera . Tarea . . . . . . . . . . . . . .

. . . .

. . . .

. . . .

. . . .

. . . .

. . . .

. . . .

. . . .

. . . .

. . . .

. . . .

. . . .

. . . .

. . . .

. . . .

. . . .

. . . .

. . . .

. . . .

. . . .

. . . .

. . . .

. . . .

. . . .

. . . .

. . . .

. . . .

. . . .

. . . .

. . . .

. . . .

. . . .

. . . .

. . . .

. . . .

. . . .

. . . .

. . . .

. . . .

. . . .

. . . .

. . . .

. . . .

. . . .

. . . .

. . . .

4. Congruencias 4.1. Congruencias . . . . . . . . . . . . . . . Tarea . . . . . . . . . . . . . . . . . . . . . . . 4.2. Sistemas residuales completos y reducidos Tarea . . . . . . . . . . . . . . . . . . . . . . . 4.3. Teoremas de Fermat, Wilson y Euler . . . Tarea . . . . . . . . . . . . . . . . . . . . . . . 4.4. Teorema snico de los residuos . . . . . . 4.5. Criterios de divisibilidad . . . . . . . . . Tarea . . . . . . . . . . . . . . . . . . . . . . . A. Indicaciones y respuestas

. . . . . . . . .

. . . . . . . . .

. . . . . . . . .

. . . . . . . . .

. . . . . . . . .

. . . . . . . . .

. . . . . . . . .

. . . . . . . . .

. . . . . . . . .

. . . . . . . . .

. . . . . . . . .

. . . . . . . . .

. . . . . . . . .

. . . . . . . . .

. . . . . . . . .

. . . . . . . . .

. . . . . . . . .

. . . . . . . . .

. . . . . . . . .

. . . . . . . . .

. . . . . . . . .

. . . . . . . . .

. . . . . . . . .

. . . . . . . . .

. . . . . . . . .

. . . . . . . . .

. . . . . . . . .

. . . . . . . . .

. . . . . . . . .

. . . . . . . . .

. . . . . . . . .

. . . . . . . . .

. . . . . . . . .

. . . . . . . . .

. . . . . . . . .

. . . . . . . . .

. . . . . . . . .

Prefacio

He aqu mi pequea contribucin a la difusin de matemticas en castellano.

III

Notacin

N Z Q R P ]a; b[ [a; b] ]a; b] [a; b[ ]a; +[ [a; +[ ] ; a[ ] ; a] x x

Los nmeros naturales {0, 1, 2, 3, . . .}. Los enteros {. . . , 3, 2, 1, 0, 1, 2, 3, . . .}. Los nmeros racionales (fracciones). Los nmeros reales. Los nmeros primos {2, 3, 5, 7, 11, . . .}. El intervalo {x R : a < x < b}. El intervalo {x R : a x b}. El intervalo {x R : a < x b}. El intervalo {x R : a x < b}. El intervalo {x R : x > a}. El intervalo {x R : x a}. El intervalo {x R : x < a}. El intervalo {x R : x a}. El nico entero que satisface x 1 < x x El nico entero que satisface x < x x + 1

Captulo

Preliminares
1.1. Buen orden
1 Denicin Se denotar los enteros mediante el smbolo Z y los nmeros naturales (enteros positivos, incluyendo al 0)

mediante el smbolo N. Un entero natural p > 1 es primo si sus nicos divisores son p mismo y la unidad 1. Se denotar el conjunto de los primos mediante el smbolo P. Si un entero diferente de 1 no es primo, entonces se dice que es compuesto. Obsrvese que 1 ni es primo ni compuesto.
2 Axioma (Axioma del buen orden) Todo conjunto no vaco de nmeros naturales contiene un elemento mnimo. 3 Ejemplo Demustrese que

2 es irracional.

a 2 es racional, esto es, que hay enteros a, b con 2 = . Esto implica que el conjunto b A = {n 2 : (n, n 2) (N \ {0})2 } no es nulo ya que contiene a a. Por el axioma del buen orden, A tiene un elemento mnimo, llmese j = k 2. Como 2 1 > 0, se tiene que j( 2 1) = j 2 k 2 = (j k) 2 es un entero positivo. Como 2 < 2 2 implica que 2 2 < 2 y tambin j 2 = 2k, se ve entonces que (j k) 2 = k(2 2) < k( 2) = j. As pues, (j k) 2 es un entero positivo de A menor que j. Esto contradice la eleccin de j como el menor elemento de A y termina la demostracin. Contrstese este mtodo con el del ejemplo 61. Resolucin: Presmase al contrario que
4 Ejemplo Sean a, b, c enteros a6 + 2b6 = 4c6 . Demustrese que a = b = c = 0.

Resolucin: Claramente basta considerar enteros positivos. Escjase un tro a, b, c que satisface la ecuacin y con m x(a, b, c) > 0 a tan pequeo como fuere posible. Si a6 + 2b6 = 4c6 entonces a ha de ser par, dgase a = 2a1 . Esto conlleva a 32a6 + b6 = 1 2c6 . Luego b es par, dgase b = 2b1 y por tanto 16a6 + 32b6 = c6 . De esto resulta que c tambin es par, dgase c = 2c1 , y 1 1 as a6 + 2b6 = 4c6 . Pero entonces m x(a1 , b1 , c1 ) < m x(a, b, c): contradiccin. As todas las variables deben ser cero. a a 1 1 1
5 Ejemplo Demustrese que el producto de n enteros consecutivos es divisible por n!.

Captulo 1

Resolucin: Obsrvese que el problema se reduce a considerar enteros estrictamente positivos, ya que si fuesen estrictamente negativos, con multiplicar por (1)n no se afecta la divisibilidad, y si incluyesen al 0, el producto sera 0, que es denitivamente divisible por n!. Premase pues que todos los enteros en consideracin son estrictamente positivos. Se utilizar el axioma del buen orden (Axioma 2) y se argir por contradiccin. Sea M el menor entero para el cual (M + 1)(M + 2) (M + n) n! no es entero. Obsrvese que M > 0 ya que n! = 1 es entero. Ahora bien, n!

(M + 1) (M + n) = (M + 1) (M + n 1)(M + n) = M(M + 1)(M + 2) (M + n 1) +(M + 2) (M + n 1)n. Por denicin de M, n! y divide a (M(M + 1)(M + 2) (M + n 1)) ((M + 2) (M + n 1)).

(n 1)! divide a n! lo cual es una contradiccin. divide a

Luego n! divide a ((M + 2) (M + n 1)n). Pero entonces

((M + 1)(M + 2) (M + n)),

Tarea
6 Problema Demustrese que no existe ningn entero en el intervalo ]0; 1[.

7 Problema (IMO, 1988) Si a, b son enteros positivos para los cuales la cantidad

a2 + b2 es un cuadrado perfecto. 1 + ab

a2 + b2 es entera, demustrese entonces 1 + ab

8 Problema Demustrese que la cantidad n3 n es siempre divisible por 6 y que n5 5n3 + 4n es siempre divisible por

120 para todo entero n.

1.2. Induccin
n + 1 S , entonces S = N.
9 Axioma (Principio (dbil) de la induccin matemtica) Si S es un conjunto de enteros tal que 0 S y si n S = 10 Axioma (Principio (fuerte) de la induccin matemtica) Si S es un conjunto de enteros tal que m S y si {m, m +

1, . . . , n} S = n + 1 S , entonces k m se tiene k S .

11 Ejemplo (USAMO, 1978) Llmese bueno al entero n si se pudiere escribir de la forma

n = a1 + a2 + + ak , en donde a1 , a2 , . . . , ak son enteros positivos, no necesariamente distintos y satisfaciendo 1 1 1 + + + = 1. a1 a2 ak Dada la informacin de que todo entero desde el 33 hasta el 73 es bueno, demustrese que todo entero 33 es bueno.

Tarea

Resolucin: Primero se demostrar que si n es bueno entonces tambin 2n + 8 y 2n + 9 son buenos. Para esto presmase que n = a1 + a2 + + ak y que 1 1 1 + + + . 1= a1 a2 ak Luego 2n + 8 = 2a1 + 2a2 + + 2ak + 4 + 4 y 1 1 1 1 1 1 1 1 + + + + + = + + = 1. 2a1 2a2 2ak 4 4 2 4 4 Adems, 2n + 9 = 2a1 + 2a2 + + 2ak + 3 + 6 y 1 1 1 1 1 1 1 1 + + + + + = + + = 1. 2a1 2a2 2ak 3 6 2 3 6 Luego si si n es bueno entonces ambos 2n + 8 y 2n + 9 son buenos. (1.1)

Sea P(n) la proposicin: todos los enteros n, n + 1, n + 2, . . . , 2n + 7 son buenos. Por hiptesis P(33) es cierta. Pero (1.1) implica la veracidad de P(n + 1) cada vez que P(n) sea cierta. Luego la asercin es demostrada por induccin fuerte.

Tarea 1.3. Principio de las pichoneras de Dirichlet


12 Regla (Principio de las casillas (pichoneras) de Dirichlet) Si n + 1 palomas vuelan hacia n pichoneras deber existir

al menos una casilla que tenga dos o ms palomas.


13 Ejemplo (Putnam, 1978) Sea A un conjunto cualquiera de 20 enteros tomados de la progresin aritmtica

1, 4, . . . , 100. Comprubese que deber de haber dos enteros distintos en A cuya suma es 104. Resolucin: Frmese una particin de los 34 elementos de la progresin en los 19 grupos {1}, {52}, {4, 100}, {7, 97}, {10, 94} . . . {49, 55}. Como se han de tomar 20 para formar el conjunto A, por el principio de las casillas de Dirichlet deben de haber dos enteros que pertenezcan a uno de los pares, y por tanto suman a 104.

Tarea
14 Problema Pngase en evidencia que entre siete enteros cualesquiera menores o iguales que 126, siempre se podr hallar

dos, llmense a y b, los cuales satisfacen las desigualdades b < a 2b.


15 Problema Dados cualesquiera 10 enteros en el conjunto {1, 2, . . . , 99} demustrese que siempre habr dos subconjuntos

disjuntos cuyos elementos sumarn a la misma suma.


16 Problema No importa que 55 enteros se elija del conjunto

{1, 2, . . . , 100}, demustrese que siempre habr dos que dieren por 10.

Captulo

Divisibilidad y primos
2.1. Divisibilidad y algoritmo de divisin
17 Denicin (Divisibilidad) Se dice que un entero d = 0 divide a otro a, denotado por da, si existe un entero d tal que

dd = a. El caso en que d no dividiere a a se denotar por d a.


18 Teorema (Propiedades de la divisibilidad) Sean a, b, c, d, x, y enteros. Entonces (da, db) = d(ax + by).

(da, ab) = db. (da, a = 0) = |d| |a|. (da, ad) = d = a. Demostracin: Se tiene que hay d , d con dd = a, dd = b. As xa + yb = xdd + ydd = (xd + yd )d, lo que implica que d(xa + yb). hay d , a con dd = a y aa = b, de donde dd a = aa = b = db.
a a es un entero no nulo, luego 1 = a d. d d por denicin da = 0. Existen d , r ambos diferentes de 0, tales que dd = a y ar = d. Entonces dd r = ar = d. Luego d r = 1 y como d , r son enteros, se tiene que d = 1, r = 1. Se colige que d = a.

el nmero

u
19 Teorema (Algoritmo de divisin) Sean b Z \ {0} y a Z. Entonces hay enteros nicos q y r tales que

a = bq + r, 0 r < |b|. 4

Divisibilidad y algoritmo de divisin a y r = a bq. En virtud de la denicin de la funcin mayor entero, b a q < q + 1, b a . luego bq a < bq + b, de donde 0 r < b = |b|. Si acaso b < 0 entonces pngase q = |b| Demostracin: Si b > 0 tmese q = Para demostrar la unicidad, supngase que a = bq + r = bq + r, con 0 r , r < |b|. Luego r r = b(q q ), de donde b(r r). Pero |r r| < |b|, lo que requiere que r = r, en virtud del Teorema 18. De aqu tambin se deduce q = q. u
20 Denicin En la ecuacin a = bq + r, a es el dividendo, b = 0 el divisor, q el cociente y r el residuo.

El algoritmo de divisin crea particiones de los enteros segn el residuo que stos dejen al ser divididos por un entero no nulo. Por ejemplo, si n = 5 el algoritmo de divisin dice que los enteros se pueden arreglar en las siguientes cinco columnas: . . . . . . 10 9 5 4 0 1 5 6 . . . . . . . . . 8 3 2 7 . . . . . . 7 2 3 8 . . . . . . 6 1 4 9 . . .

El arreglo aqu mostrado evidencia que los enteros vienen en uno de cinco sabores: aquellos cuyo residuo es 0 al ser divididos por 5, aquellos cuyo residuo es 1 al ser divididos por 5, etc. Dicho de otra manera, todo entero es de la forma 5k, 5k + 1, 5k + 2, 5k + 3 5k + 4. Obsrvese adems que se puede decir que todo entero es de la forma 5k, 5k 1 5k 2. El algoritmo de divisin pues discrimina y crea clases entre los enteros, llamadas clases de equivalencia, que se denotarn (en el caso cuando el divisor es 5) por 5Z = {. . . , 15, 10, 5, 0, 5, 10, 15, . . .} = 0, 5Z + 1 = {. . . , 14, 9, 4, 1, 6, 11, 16, . . .} = 1, 5Z + 2 = {. . . , 13, 8, 3, 2, 7, 12, 17, . . .} = 2, 5Z + 3 = {. . . , 12, 7, 2, 3, 8, 13, 18, . . .} = 3, 5Z + 4 = {. . . , 11, 6, 1, 4, 9, 14, 19, . . .} = 4, y se pondr Z5 = {0, 1, 2, 3, 4}.
21 Ejemplo Demustrese que el cuadrado de todo entero es de la forma 4k o de la forma 4k + 1. Luego demustrese que ningn entero en la sucesin 11, 111, 1111, 11111, . . .

es el cuadrado de un entero. Resolucin: Si el entero es par, es decir de la forma 2a, su cuadrado es (2a)2 = 4a2 , que es de la forma 4k. Si el entero es impar, digamos 2t + 1, entonces (2t + 1)2 = 4(t2 + t) + 1, que es de la forma 4k + 1. Ahora bien, para n 2, 11 . . . 1 = 11 . . . 11 00 + 8 + 3 = 100 11 . . . 11 +8 + 3 := 4s + 3,
n1 s n2 1 s n2 1 s

en donde s = 25 11 . . . 11 +2. As pues, todo nmero en esta sucesin es de la forma 4k + 3. Pero se sabe que un cuadrado
n2 1 s

ha de tener la forma 4k 4k + 1 y por lo tanto ningn miembro de esta sucesin es el cuadrado de un entero.

6
22 Ejemplo Sea n > 0 entero.

Captulo 2

Sea a = 1. Demustrese la identidad 1 + a + a2 + + an1 = Demustrese la identidad


xn yn = (x y)(xn1 + xn2 y+ xn3 y2 + + xyn2 + yn1 ). (2.2)

an 1 . a1

(2.1)

De esto se deduce que si x, y son enteros con x = y entonces x y divide a x y . Si n es impar, hgase patente que (x + y)(xn + yn ). Demustrese que si k es un entero positivo impar 1k + 2k + + nk es divisible por 1 + 2 + + n. Resolucin: Para el primer inciso, se proceder por induccin. Para n = 1 es claro que 1 = 1 + a + a2 = a1 , y para n = 2 es evidente que a1

a3 1 . Suponiendo la validez de 2.1 para n, habr de demostrarse para n + 1. Ahora bien a1 (1 + a + a2 + + an1 ) + an = = = an 1 + an a1 an 1 + an+1 an a1 an+1 1 , a1

demostrando la validez de 2.1 para n + 1. As la primera asercin queda demostrada por induccin. Para demostrar 2.2 basta poner a =
x en 2.1 y simplicar. Es evidente entonces que (x y)(xn yn ). y

Si n fuere impar, entonces (y)n = yn y con substituir y por y en 2.2 se obtiene el resultado. Para obtener la ltima aseveracin obsvese primero que 1 + 2 + + n = n(n + 1) . 2

Se considerar los casos cuando n es par y cuando n es impar por separado. Presmase primero que n es par. Luego
n 2

es un entero y cada una de las expresiones n2 2


k

1k + (n 1)k ; 2k + (n 2)k ; ; es divisible por


n 2

n+2 2

n 2

; nk ,

en vista del inciso anterior. Reagrupando de la manera 1k + nk ; 2k + (n 1)k ; ; n 2


k

n+2 2

tambin se ve que la suma es divisible por n + 1. Como n(n + 1) divisible por . 2

n 2

y n + 1 no tienen factores en comn, se deduce que la suma es

Divisibilidad y algoritmo de divisin Presmase ahora que n es impar, de donde


n+1 2

7 es un entero. Cada una de las expresiones n1 2


k

1k + nk ; 2k + (n 1)k ; ; es divisible por


n+1 2

n+3 2

n+1 2

en vista del inciso anterior. De igual manera la suma es divisible por n ya que cada una de las expresiones 1k + (n 1)k ; 2k + (n 2)k ; ; n1 2
k

n+1 2

; nk ,
n(n+1) . 2

lo es. Como

n+1 2

y n no tienen factores en comn, se deduce que la suma es divisible por

23 Ejemplo Demustrese que si n es un entero positivo tal que 2n + 1 es un cuadrado, entonces n + 1 es la suma de dos cuadrados consecutivos.

Resolucin: Como 2n + 1 es un cuadrado impar, tenemos 2n + 1 = (2t + 1)2 para algn entero t. Resolviendo para n, n= (2t + 1)2 1 = 2t2 + 2t. 2

Luego n + 1 = t2 + (t + 1)2 , la suma de dos cuadrados consecutivos.


24 Ejemplo Demustrese que el nico primo de la forma n4 + 4 es el 5.

Resolucin: Se puede restringir el argumento a enteros positivos. Obsrvese que n4 + 4 = n4 + 4n2 + 4 4n2 = (n2 + 2)2 (2n)2 = (n2 2n + 2)(n2 + 2n + 2).

Si este producto es un nmero primo entonces el factor ms pequeo debe ser igual a 1. As n2 2n + 2 = 1, o sea (n 1)2 = 0, esto es n = 1. As, el nico primo de esta forma es 14 + 4 = 5.
25 Ejemplo Sean a1 , a2 , . . . , a2n enteros tales que la ecuacin

(x a1 )(x a2 ) (x a2n ) (1)n (n!)2 = 0 posee una solucin entera a. Demustrese que a= a1 + a2 + + a2n . 2n

Resolucin: Evidentemente se tiene a = ai para ninguna de las i y los enteros a ai son 2n enteros diferentes. As pues
2 (a a1 )(a a2 ) (a a2n ) (1)(2) (n)(1)(2) (n) = (n!) ,

la igualdad ocurriendo si y slo si {a1 , a2 , . . . , a2n } = {1, 2, . . . , n, 1, 2, . . . , n}. Se deber tener entonces (a a1 ) + (a a2) + + (a a2n ) = 1 + 2 + + n 1 2 n s y slo si s y slo si 2an (a1 + a2 + + a2n ) = 0 a= como se quera demostrar. a1 + a2 + + a2n , 2n

Captulo 2

Tarea
26 Problema Hallse todos los enteros positivos de la forma

1 r+ , r donde r es un nmero racional.


27 Problema Demustrese que el entero

11 . . . 11
221 1 s

es compuesto.
28 Problema Demustrese que 100 divide a 1110 1. 29 Problema Demustrese que entre tres enteros siempre se pueden escoger dos tales que a3 b ab3 sea divisible por 10. 30 Problema Demustrese, va induccin, que la expresin

33n+3 26n 27 es un mltiplo de 169 para todos los nmeros naturales n.


31 Problema Demustrese que si 3n + 1 es un cuadrado, entonces n + 1 es la suma de tres cuadrados. 32 Problema Demustrese que si n > 11 entonces n se puede escribir como la suma de dos nmeros compuestos. 33 Problema (AHSME, 1976) Sea r el residuo cuando 1059, 1417 y 2312 se dividen por d > 1. Halle el valor de d r. 34 Problema Demustrese que n2 + 23 es divisible por 24 para un nmero innito de nmeros n. 35 Problema La suma de enteros positivos es 1996. Cul es el valor mximo de su producto? 36 Problema (Eotvos, 1899) Comprubese que para todo entero positivo n, la expresin

2903n 803n 464n + 261n es siempre divisible por 1897.


37 Problema Demustrese que todo entero n > 6 puede ser escrito como la suma de dos enteros ambos mayores que 1 tales que cada sumando sea relativamente primo. 38 Problema Demustrese que si ambos p es primo, o bien 8p 1 es primo y 8p + 1 compuesto o viceversa. 39 Problema Demustrese que 7 divide a 22225555 + 55552222. 40 Problema Demustrese que si 2n 1 es un nmero primo, entonces n es un nmero primo. Primos de esta forma se llaman

primos de Mersenne.
41 Problema Demustrese que si 2n + 1 es un nmero primo, entonces n es una potencia de 2. Primos de esta forma se

llaman primos de Fermat.

Mximo comn divisor


42 Problema ((UM)2 C4 , 1987) Dado que 1002004008016032 tiene un factor primo p > 250000, encuntrese. 43 Problema Demustrese que

a3 + b3 + c3 3abc = (a + b + c)(a2 + b2 + c2 ab bc ca). Luego demustrese que


6(a + b + c) = 6(a3 + b3 + c3 ). Demustrse adems que que si n es la suma de tres cubos consecutivos, entonces 9n.
44 Problema Comprubese que el producto de cuatro enteros consecutivos, diferentes de 0, jams es un cuadrado. 45 Problema Demustrese que si k es impar entonces 2n+2 divide a

k2 1 para todos los enteros naturales n 1.

a y b. Si d es un divisor comn de a y b tal que cualquier otro divisor comn de a y b divide a d, entonces d es el mximo comn divisor de a y b, denotado por MCD (a, b). Obsrvese que esto requiere que d sea > 0.
47 Denicin Dcese que dos enteros a y b son relativamente primos si MCD (a, b) = 1. 48 Teorema (Teorema de Bachet-Bzout) Sea (a, b) Z2 , (a, b) = (0, 0). Si d = MCD (a, b) entonces hay enteros x, y

46 Denicin (Mximo comn divisor) Sea (a, b) Z2 , (a, b) = (0, 0). Si ra y rb entonces r es un divisor comn de

2.2. Mximo comn divisor

tales que

ax + by = d. Demostracin: Considrese el conjunto S = {n Z : n > 0, n = as + bt, (s, t) Z2 }. S = ya que o bien a S o bien b S . Luego por el buen orden de los enteros, S tiene un elemento mnimo estrictamente positivo al que se llamar n0 = ax0 + by0 . Obsrvese que dn0 en virtud del Teorema 18 ya que da y db. Ahora bien, por el algoritmo de divisin existen enteros q, r con a = qn0 + r, 0 r < n0 . Si r = 0 entonces r = a qn0 = a q(ax0 + by0 ) = a(1 qx0) qby0 S que es menor que n0 , contradiciendo la denicin de n0 , de donde se concluye que r = 0. De la misma manera se puede demostrar que n0 b. Luego n0 es un divisor comn de a y b, por lo tanto divide a d. Ya que dn0 y n0 d se tiene, en virtud del Teorema 18, se tiene que d = n0 . Como ambos d > 0, n0 > 0 se colige que d = n0 . u
49 Ejemplo Si MCD (a, b) = 1 entonces MCD a + b, a2 ab + b2 = 1 3.

Resolucin: Sea d = MCD a + b, a2 ab + b2 . Ahora bien, d divide a (a + b)2 a2 + ab b2 = 3ab.

10

Captulo 2

As, d divide a 3b(a + b) 3ab = 3b2 . De manera semejante se deduce que d3a2 . Pero entonces d MCD 3a2 , 3b2 = 3 MCD a2 , b2 = 3 MCD (a, b)2 = 3.
50 Ejemplo Sean m, n, a = 1 enteros positivos. Demustrese que

MCD (am 1, an 1) = aMCD(m,n) 1. Resolucin: Pngase d = MCD (m, n), sd = m, td = n. Entonces am 1 = (ad )s 1 es divisible por ad 1 y de

manera semejante, an 1 es divisible by ad 1. As (ad 1) MCD (am 1, an 1). Ahora bien, en virtud al Teorema de Bachet-Bezout (Teorema 48) existen enteros x, y con mx + ny = d. Ntese que x, y habrn de tener signos opuestos (no pueden ser ambos negativos, ya que d sera entonces negativo. Si ambos fuesen positivos entonces d m+n, lo que contradice al hecho que d m, d n). Presmase pues, sin prdida de generalidad, que x > 0, y 0. Pngase t = (am 1, an 1). Entonces t(amx 1) y t(any 1). Luego t((amx 1) ad (any 1)) = ad 1, estableciendo el resultado.

Tarea
51 Problema (IMO, 1959) Comprubese que la fraccin

21n + 4 es irreducible para todo entero natural n. 14n + 3

52 Problema (AIME, 1985) Los nmeros de la sucesin

101, 104, 109, 116, . . . son de la forma an = 100 + n2, n = 1, 2, . . .. Para cada n pngase dn = MCD (an , an+1 ). Hllese m x dn . a
n1

53 Problema Tmese cualesquiera 51 enteros de entre 1, 2, . . . , 100. Demustrese que hay al menos dos que son relativamente

primos.
54 Problema Demustrese que el producto de tres enteros estrictamente positivos consecutivos jams ser una potencia per-

fecta.

2.3. Primos y factorizacin nica


55 Lema (Lema de Gauss) Si dab y MCD (d, a) = 1 entonces db.

Demostracin: Por el Teorema de Bachet-Bezout (Teorema 48) existen enteros x, y tales que ax + dy = 1. Luego bax + bdy = b. Como dab se tiene que d(bax + bdy) = b. u
56 Lema (Lema de Euclides) Si p es primo, y pab o bien pa o bien pb.

Demostracin: Si p a entonces MCD (p, a) = 1. Gracias al Lema de Gauss (Lema 55) se tiene que pb. u
57 Teorema (Teorema fundamental de la aritmtica) Todo entero n > 1 puede descomponerse en factores de la manera

n = p1 1 pr , r en donde p1 < p2 < < pr son primos. Esta representacin es nica, a la cual se llamar la factorizacin cannica de n..

Primos y factorizacin nica Demostracin: Si n > 1 es primo, entonces no hay nada que demostrar. Supngase que n > 1 es el menor entero no primo que no se puede descomponer de la manera dicha. Como n no es primo, existen n > 1, n > 1 con n = n n . Pero luego n y n son menores que n y por tanto se pueden descomponer como n = q1 1 qs y s 1 t n = q1 qt donde algunos de los exponentes pueden ser 0. Se sigue que n = n n = n = q1 1
+1 2 +2 q2

11

lo que contradice la suposicin de que n no se poda descomponer en primos. Para demostrar la unicidad de la descomposicin, se argir por induccin. Supngase que todo entero mayor que 1 y menor que n puede descomponerse en primos de manera nica. Si n = p1 1 pr = q1 1 qs , r s en donde p1 < p2 < < pr y q1 < q2 < < qr . Entonces por el Lema de Euclides (Lema 56) p1 debe dividir a exactamente una a de las qs y q1 debe dividir a exactamente una de las ps. Pero esto fuerza p1 = q1 . Luego al dividir por p1 uno y otro lado se obtiene se obtiene n 1 1 = p1 1 pr = p1 1 q2 2 qs . r s p1 n Por la hiptesis de induccin se descompone en primos de manera nica, luego r = s, pi = qi y 1 1 = p1 1 1, 2 = 2 , . . . , r = r , de donde se colige que 1 = 1 y por lo tanto n tambin se descompone en primos de manera nica. u
58 Teorema (Euclides-Innitud de los primos) El conjunto P de los primos es inagotable. An ms, si p1 = 2, p2 = 3, . . . y

en general pk es el k-simo primo, entonces pk+1 p1 pk + 1. Demostracin: Considrese el entero n = p1 pk + 1. Por el Teorema fundamental de la aritmtica (Teorema 57) o bien n = p1 pk + 1 es primo, o puede descomponerse como un producto de primos, lo que demuestra la existencia de una lista de primos dividiendo a n, lista que se llamar P. Ahora bien, al dividir n por los primos de la lista P = {p1 , p2 , . . . , pk }, n deja residuo 1. De aqu se inere que P P = y luego a cualquier lista nita de primos puede agregrsele un primo ms, de donde ninguna lista nita de primos es exhaustiva. El primo mnimo p diviendo a n debe ser mayor que pk y se tiene p n en virtud del Teorema 18. Es claro que pk+1 p. u La sucesin de los primos comienza pues as 2, 3, 5, 7, 11, 13, 17, 19, 23, 29, 31, 37, 41, 43, 47, 53, . . .
59 Denicin Un mltiplo comn de dos enteros a, b es un entero no negativo que es divisible por ambos a y b. Si m es

mltiplo comn de a, b y si m divide a todo otro mltiplo comn, entonces m es el mnimo comn mltiplo, de a, b, al que se denotar por mcm (a, b).
60 Teorema Sean n = q1 1 qs y n = q1 1 qt t donde algunos de los exponentes pueden ser 0. Entonces s

MCD n, n y An ms: y si k es un entero positivo entonces mcm n, n


= q1

mn (1 ,1 ) mn (2 ,2 )

q2

(2.3) (2.4) (2.5)

= q1

m x (1 ,1 ) m x (2 ,2 ) a a

q2

nn = MCD n, n mcm n, n . MCD (a, b)k = MCD ak , bk , mcm (a, b)k = mcm ak , bk .

(2.6)

12

Captulo 2 Demostracin: Las dos primeras aserciones son evidentes gracias al Teorema fundamental de la aritmtica (Teorema 57). Para la tercera asercin basta notar que mn(, ) + m x(, ) = + . a La cuarta se deriva de las primeras dos. u

61 Ejemplo Demustrese que

2 es irracional.

Resolucin: Supngase a miras de contradiccin que 2 = a/b con enteros positivos a, b relativamente primos. Entonces 2b2 = a2 . El lado siniestro de esta ecuacin tiene un nmero impar de factores primos mientras que el diestro tiene un nmero par. Esto contradice el Teorema fundamental de la aritmtica (Teorema 57). Contrstese este mtodo con el del ejemplo 3.
62 Ejemplo Demustrese que dados cualesquiera 33 enteros diferentes con todos sus factores primos en el conjunto {5, 7, 11, 13, 23},

siempre hay dos distintos cuyo producto es un cuadrado. Resolucin: Cada uno de los 33 enteros es de la forma 5a 7b 11c 13d 23f . As, a cada uno de los 33 enteros se les puede asociar un vector de la forma (a, b, c, d, f). Dependiendo la paridad de los componentes del vector, hay 32 = 25 tipos de estos vectors. Por ejemplo, uno de los tipos es (par, par, impar, impar, par). Pinsese de estas 32 clases de vectores como en 32 casillas. A los 33 enteros se les distribuir en las 32 casillas y por tanto, una de las casillas tendr al menos dos enteros diferentes. En esta casilla los exponentes de los nmeros son de la misma paridad, as que al multiplicarse, todos los exponentes sern pares. Luego este producto ser un cuadrado.

Tarea
63 Problema (IMO 1985) Dado un conjunto M de 1985 enteros positivos distintos, ninguno de cuyos factores primos es

mayor que 26, demustrese que M siempre tendr cuatro elementos distintos cuyo producto es una cuarta potencia.
64 Problema Tmese 51 enteros cualesquiera del conjunto

{1, 2, . . . , 100}. Demustrese que siempre habr dos, uno dividiendo al otro.
65 Problema Demustrese que si el polinomio

p(x) = a0 xn + a1 xn1 + + an1 x + an con coecientes ntegros alcanza el valor de 7 para cuatro valores ntegros de x entonces no puede tomar el valor de 14 para ningn valor ntegro de la variable x.
66 Problema Comprubese que m5 + 3m4 n 5m3 n2 15m2 n3 + 4mn4 + 12n5 nunca ser igual a 33. 67 Problema Demustrese que la suma

S = 1/2 + 1/3 + 1/4 + + 1/n jams es ntegra.


68 Problema Demustrese que existe un entero nico n para el cual 28 + 211 + 2n es un cuadrado perfecto.

Algoritmo de Euclides y ecuaciones diofnticas lineales

13

2.4. Algoritmo de Euclides y ecuaciones diofnticas lineales


Se dar ahora un procedimiento ecaz para hallar soluciones x, y a la ecuacin MCD (a, b) = ax + by. Este procedimiento se llama el algoritmo de Euclides y opera como sigue. Sean a, b enteros positivos. Luego de aplicar el algoritmo de divisin repetidamente, se ve que a = bq1 + r2 , 0 < r2 < b, b = r2 q2 + r3 0 < r3 < r2 , r2 = r3 q3 + r4 0 < r4 < r3 , (2.7) . . . . . . . . . . . . rn2 rn1 = rn1 qn1 + rn = rn qn . 0 < rn < rn1 ,

La sucesin de residuos eventualmente llegar a un rn+1 que ser 0 ya que la sucesin b, r2 , r3 , . . . es una sucesin de enteros montona decreciente la cual no puede tener ms de b trminos estrictamente positivos.
69 Lema Si b = 0, a = qb + r entonces MCD (a, b) = MCD (b, r).

Demostracin: Pngase d = MCD (a, b) , c = MCD (b, r) . Como da y db, se sigue que d(aqb) = r. As pues d es un divisor comn de r y b. Esto implica que dc. Por otra parte, cr y cb implican que c(qb+r) = a. Luego c es un divisor comn de a y b de donde cd, lo que acaba la demostracin. u
70 Teorema Si rn es el ltimo residuo diferente de 0 encontrado en el algoritmo de Euclides, entonces

rn = MCD (a, b) . Demostracin: Aplicando el Lema 69 varias veces se ve que MCD (a, b) = = = = = u MCD (b, r2 ) MCD (r2 , r3 ) . . . MCD (rn1 , rn ) rn .

 De las ecuaciones en 2.7 se ve que


r2 r3 r4 . . . rn = = = . . . a bq1 b r2 q2 r2 r3 q3 . . .

= rn2 rn1 qn1 ,

luego es posible expresar MCD (a, b) como una combinacin lineal de a y b trazando estas igualdades en reversa.
71 Teorema Sean a, b, c son enteros tales que (a, b)|c. Dada una solucin (x0 , y0 ) de la ecuacin diofntica lineal

ax + by = c, cualquier otra solucin es de la forma b a x = x0 + t , y = y0 t , d d en donde d = (a, b) y t Z.

14

Captulo 2 Demostracin: Es inmediato que si (x0 , y0 ) es una solucin de ax + by = c, entonces x = x0 + tb/d, y = y0 ta/d es tambin una solucin. Demostrarse ahora que toda otra solucin es de la forma deseada. Supngase que (x , y ) satisface ax + by = c. Si adems ax0 + by0 = c entonces a(x x0 ) = b(y0 y ). Dividiendo por d = (a, b), a b (x x0 ) = (y0 y ). d d a Como (a/d, b/d) = 1, se sigue que |(y0 y ), por virtud del Lema de Euclides. As pues existe un entero t d a tal que t = y0 y , sto es, y = y0 ta/d. Se colige entonces que d a b a (x x0 ) = t , d d d which is to say x = x0 + tb/d, demostrando el teorema. u

72 Ejemplo Hllese MCD (23, 29) mediante el algoritmo de Euclides y Encuntrese soluciones enteras para la ecuacin

23x + 29y = 1. Resolucin: Se tiene 29 = 1 23 + 6, 23 = 3 6 + 5, 6 = 1 5 + 1, 5 = 5 1.

El ltimo residuo no nulo es el 1 de donde MCD (23, 29) = 1. Ahora bien,

Luego, 1 = = = = =

6 = 29 1 23. 615 6 1 (23 3 6) 4 6 1 23 4(29 1 23) 1 23 4 29 5 23.

5 = 23 3 6,

1 = 6 1 5,

Esto resuelve la ecuacin con x = 5, y = 4.

73 Ejemplo Encuntrese un nmero innito de soluciones para 23x + 29y = 1.

Resolucin: Gracias al ejemplo 72 se tiene que el par x0 = 5, y0 = 4 es una solucin. Se puede encontrar una familia innita de soluciones en poniendo x = 5 + 29t, y = 4 23t, t Z.
74 Ejemplo Hllese soluciones enteras para la ecuacin 23x + 29y = 7.

Resolucin: Del ejemplo 72 se tiene 23(5) + 29(4) = 1. Multiplicando uno y otro lado por 7, 23(35) + 29(28) = 7, lo que resuelve el problema.

Tarea

15

Tarea

Captulo

Funciones aritmticas
3.1. Funciones multiplicativas
75 Denicin (Funcin aritmtica) Llmase funcin aritmtica a una funcin f : N\{0} C. f es multiplicativa si MCD (m, n) = 1 = f(mn) = f(n)f(n). f es completamente multiplicativa si f(mn) = f(n)f(n) para todos los enteros positivos m y n.

 Toda funcin completamente multiplicativa es, a fortiori, multiplicativa.

Una funcin multiplicativa est completamente determinada por sus valores en las potencias de los primos. As, si n = p1 1 pr entonces r 1 r 2 f(n) = f(p1 ) f(pr ). Adems f(1 1) = f(1)f(1) = (f(1)) . Luego, o bien f(1) = 1 o bien f(1) = 0. Este ltimo caso hace que la funcin sea idnticamente 0 por lo cual no se considerar. Por tanto, se tendr por convencin que para toda funcin multiplicativa f, f(1) = 1.

76 Teorema Sea n > 0 un entero y f, g funciones con

g(n) =

dn

f(d).

Si f es multiplicativa, tambin lo es g. Demostracin: Tmese adems n > 0 entero, con MCD n, n d = d1 d2 , d1 n, d2 n , de donde MCD (d1 , d2 ) = 1. Luego g(nn ) =
dnn d1 n, d1 n,

= 1. Todo dnn puede descomponerse en

f(d)

d2 n d2 n

f(d1 d2 )

f(d1 )(d2 )

d1 n

f(d1 )

d2 n

f(d2 )

16

Funciones multiplicativas demostrando la asercin. u


77 Denicin (Funcin nmero de divisores) La funcin

17

N \ {0} d: n cuenta el nmero de divisores positivos de un entero positivo n.


78 Teorema La funcin

N \ {0} 1
dn

N \ {0} d: n

N \ {0} 1
dn

es multiplicativa, y si n = p1 1 pr es la descomposicin cannica en primos de n, entonces r d(n) = (1 + 1) (r + 1). Demostracin: Ya que la funcin d 1 es completamente multiplicativa es, a fortiori, multiplicativa. Luego d(n) = 1 es multiplicativa en virtud del Teorema 76. Si p es primo, p tiene +1 divisores: 1, p, p2 , . . . , p .
dn

As d(p ) = + 1 y d(n) = d(p1 1 ) d(pr ) = (1 + 1) (r + 1). r u


79 Ejemplo (El problema de los casilleros) La guardarropa de un gimnasio tiene 100 casilleros y 100 usuarios. Al princi

pio, todos los casilleros estn abiertos. Entra el usuario nmero 1 y cierra todos los casilleros. Entra el usuario nmero 2, y cierra todos los casilleros con nmero par. Entra el usuario nmero 3 y cambia de estado (de cerrado a abierto o vice versa) todos los casilleros cuyo nmero es un mltiplo de 3. Entra el usuario nmero 4 y cambia de estado (de cerrado a abierto o vice versa) todos los casilleros cuyo nmero es un mltiplo de 4. Sucede as sucesivamente hasta que entra el usuario nmero 100 y cambia de estado el casillero nmero 100. Qu casilleros permanecen cerrados? Resolucin: Se ver que los casilleros cuyo nmero es un cuadrado perfecto permanecen cerrados. As los casilleros nmero 1, 4, 9, 16, 25, 36, 49, 64, 81 y 100 son los que permanecen cerrados. Obsrvese que el casillero nmero n es afectado por el usuario d si y slo si d divide a n. As slo aquellas n que tengan un nmero impar de divisores permaneceran cerradas. n n Ahora bien, cada factor d de n se puede aparear con , y as, n tendr un nmero impar de factores si y slo si se tiene d = , d d esto es, si n es un cuadrado perfecto.
80 Denicin (Funcin suma de divisores) La funcin

N \ {0} N \ {0} d : n
dn

suma los divisores positivos de un entero positivo n.

18
81 Teorema La funcin

Captulo 3

N \ {0} N \ {0} d : n
dn

es multiplicativa y si n = p1 1 pr es la descomposicin cannica en primos de n, entonces r (n) = p1 1 1 pr +1 1 r . p1 1 pr 1


+1

Demostracin: Ya que la funcin d d es completamente multiplicativa es, a fortiori, multiplicativa. Luego d es multiplicativa en virtud del Teorema 76. Si p es primo, p tiene por suma de divisores la suma (n) =
dn

geomtrica (p ) = 1 + p + p2 + + p = Luego (n) = (p1 1 ) (pr ) = r u


82 Denicin Dado n = p1 1 pr , la funcin : N \ {0} N cuenta cuntos primos distintos n tiene sin contar factores r
+1

p+1 1 . p1

p1 1 1 pr +1 1 r . p1 1 pr 1

repetidos, esto es (n) = r. La funcin : N \ {0} N cuenta cuntos primos distintos n tiene incluyendo factores repetidos, esto es (n) = 1 + 2 + + r . Es evidente que tanto como son multiplicativas, an ms, es totalmente multiplicativa. Tambin es evidente que (n) (n) siempre y que (n) = (n) si y slo si n no tiene ningn factor cuadrado mayor que 1.

83 Denicin (Funcin de Mbius) Sean p, p1 , . . . , pr primos distintos y

n > 0 entero. La funcin : se dene como sigue: N \ {0} {1, 0, 1} n (n) si n = 1 si (n) = (n) si (n) > (n)

1 (n) = (1)(n) 0

84 Teorema La funcin de Mbius es multiplicativa.

Demostracin: Sea n = ab, con MCD (a, b) = 1. Si p P y p2 n entonces p2 debe dividir o bien a a o bien

a b. En todo caso (n) = (a)(b) = 0. Si a = p1 pr , b = q1 qs donde todas las ps y qs son diferentes, entonces (n) = (1)r+s = (1)r (1)s = (a)(b), de donde es multiplicativa. u

Funciones multiplicativas
85 Teorema Si n > 0 es un entero,

19

dn

(d) =

1 si 0 si

n=1 n>1

Demostracin: Pngase A = dn : (d) = k, p2 d, p P . Entonces #(A) = n y si d A se tiene (d) = (1)k . La suma requerida es k
(n)

dn

(d) =
k=0

(n) (1)k . k

En virtud del Teorema del binomio, (1 1)(n) = 0. u


86 Teorema (Frmula de inversin de Mbius) Sean f y F funciones aritmticas, en donde F(n) =

dn

f(d). Luego

f(n) =

dn

(d)F(n/d) =

dn

(n/d)F(d).

Recprocamente, si f(n) =

dn

(d)F(n/d) para todo entero n > 0, entonces F(n) =

dn

f(d).

Demostracin: Se tiene
dn

(d)F(n/d) =

dn dn s n d dsn sn

f(s)

(d)f(s)

f(s)

d n s

(d).

Gracias al Teorema 85, la suma interna es diferente de 0 slo cuando en la suma externa el de s = n, que simplica a f(n).

n s

= 1. As, el nico trmino que sobrevive

20 Recprocamente, si
dn

Captulo 3

f(d) =

dn sd dn sd sn r n s sn

(s)F(d/s)

(d/s)F(s)

(r)F(s)

F(s)

r n s

(r).

Utilizando el Teorema 85, la suma interna ser 0 a menos que s = n, en cuyo caso la suma simplica a F(n). u
87 Denicin (Funcin indicatriz de Euler) La funcin

cuenta el nmero de enteros entre 1 y n que son relativamente primos a n:

N \ {0} N \ {0} n (n)

(n) = #(k : 1 k n, MCD (k, n) = 1}.


88 Teorema Sea n 1 entero. Entonces
dn

(d) = n.

Demostracin: Para cada divisor d de n, sea Td (n) el conjunto de enteros positivos n cuyo mximo comn divisor con n es d. Tanto d vara sobre los divisores de n, los conjuntos Td (n) forman una particin de {1, 2, . . . , n} y por lo tanto
dn

Td (n) = n.

Se demostrar de inmediato que Td (n) posee (n/d) elementos. Obsrvese que los elementos de Td (n) yacen n en el conjunto d, 2d, . . . d. Si k Td (n), entonces k = ad, 1 a n/d y MCD (k, n) = d. Pero entonces d k n n MCD , = 1, lo que implica que MCD a, = 1. Se deduce que contar los elementos de Td (n) es d d d n = 1. Pero el nmero de equivalente a contar el nmero de enteros a que satisface 1 a n/d, MCD a, d estos enteros es precisamente (n/d). Colegimos que n=
dn

(n/d).

Pero en tanto d vara sobre los divisores de n,

n vara en reverso. As pues n = (n/d) = (d), lo que d


dn

dn

demuestra el teorema.u

Tarea
89 Corolario Si n 1 es un entero,

21

(n) = n

dn

(d) . d

Demostracin: Esto se deduce de inmediato en combinando el Teorema de inversin de Mbius (Teorema 86) y el Teorema 88. u
90 Corolario La funcin

es multiplicativa y si n = p1 1 pr es la descomposicin cannica en primos de n, entonces r (n) =


N \ {0} N \ {0} n (n)


(p p1 ) = n

1 . p

p n

pn

Demostracin: Combnese el Corolario 89 con el Teorema 76 para demostrar que es multiplicativa. Por otra parte, si p es primo, hay p1 enteros positivos p que tienen un factor comn con p: 1p, 2p, 3p, . . . , p1 p. Por lo tanto (p ) = p p1 de donde resulta la segunda aseveracin. u

Tarea
91 Problema Demustrese que si n es compuesto entonces (n) n

n. Cuando se vericar la igualdad?

92 Problema (AIME, 1992) Encuntrese la suma de todos los nmeros racionales positivos que sean menores que 10 y tengan denominador 30 cuando se escriban en trminos mnimos.. 93 Problema Demustrese que (n) n2(n) . 94 Problema Demustrese que (n) >

n para n > 6.

95 Problema Demustrese que si (n)n, entonces n debe de ser de la forma 2a 3b para enteros no negativos a, b. 96 Problema Demustrese que si (n)n 1, entonces n no es divisible por ningn cuadrado mayor que 1.

97 Problema (Mandelbrot 1994) Cuatrocientas personas se colocan alrededor de un crculo. Se marca a una persona, se

perdona a las prximas k personas, luego se marca a otra, se perdona a las prximas k, etctera, continuando hasta que se marque a una persona por segunda vez. Para cuntos valores de k menores que 400 se marcarn a todas las personas en el crculo al menos una vez?
98 Problema Demustrese que si (n)n 1 y n es compuesto, entonces n tiene al menos tres factores primos distintos. 99 Problema Demustrese que si (n)n 1 y n es compuesto, entonces n tiene al menos cuatro factores primos distintos.

100 Problema Descrbase todos los enteros positivos n para los cuales d(n) = 10.

22
101 Problema Demustrese que

Captulo 3

d(2n 1) d(n).
102 Problema Demustrese que d(n)

3n vericndose la igualdad si y slo si n = 12.

103 Problema Demustrese que se cumple la expansin


n=1

d(n)tn =

n=1

tn , 1 tn

llamada expansin de Lambert.


104 Problema Pngase d1 (n) = d(n), dk (n) = d(dk1 (n)), k = 2, 3, . . .. Descrbase dk (n) para k lo sucientemente

grande.
105 Problema Dado m N, demustrese que el conjunto

A = {n N : md(n)} posee una progresin aritmtica innita.


106 Problema Sea n un nmero perfecto. Demustrese que

1
d dn

= 2.

107 Problema Demustrese que


dn

d = nd(n)/2 .

108 Problema (AIME 1995) Sea n = 231 319 . Cuntos divisores positivos de n2 son menores que n pero no dividen a n? 109 Problema Demustrese que si n es compuesto, entonces (n) > n +

n.

110 Problema Demustrese que la ecuacin (n) = n + k, donde k > 1 es un nmero natural jo, posee un nmero nito de

soluciones.
111 Problema Caractercese todos los enteros positivos n para los cuales (n) es impar. 112 Problema Demustrese que p es primo si y solamente si (p) = 1 + p. 113 Problema Demustrese que

(n!) 1 1 1 + + + . n! 2 n
114 Problema Demustrese que ninguna potencia de un primo no puede ser un nmero perfecto. Luego, demustrese que de

existir un nmero perfecto impar, ste debe tener al menos dos factores primos distintos.

La funcin parte entera

23

115 Problema Demustrese que si n es un nmero perfecto impar, entonces solamente uno de sus factores primos ocurre con

potencia par.
116 Problema Demustrese que
n n

(k) =
k=1 j=1

n . j

117 Problema Hllese todos los conjuntos de enteros positivos {a, b, c} tales que a b c = 462.

3.2. La funcin parte entera


118 Denicin Sea x R. Llmase suelo de x (o funcin mayor entero de x) al nico entero x que satisface

x 1 < x x. Ntese que cada x R puede escribirse como x = x + {x}, en donde 0 {x} < 1 es la parte fraccionaria de x. Anlogamente, llmase techo de x (o funcin menor entero de x) al nico entero x que satisface x x < x+1
119 Teorema Sean (, ) R2 , a Z, n N \ {0}. Entonces

1. 2. 3.

+a = +a = n n

+ + + +1 Demostracin: Se demostrar cada inciso individualmente. 1. Sea m = + a . Entonces m + a < m + 1. De aqu m a < m a + 1. Esto signica que m a = , completando la demostracin de este inciso. 2. Pngase /n = /n + , 0 < 1. Como n /n es un entero, se deduce del inciso 1 que = n /n + n = n /n + n . Ahora bien, 0 n n < n, y por lo tanto, 0 n /n < 1. Si = n /n, entonces se obtiene = + , 0 < 1, n n demostrando este inciso. 3. De las desigualdades 1 < , 1 < se obtiene + 2 < + + . Ya que + es un entero + , entonces ser + . De aqu se colige que + + . Adems + es + + 2, de donde su parte ntegra + deber ser + + 2, pero como + < + + 2 entonces se tiene + + + 1, demostrando las desigualdades. u

24
120 Teorema (Formula de De Polignac) La mxima potencia del primo p que divide a n! est dada por
k=1

Captulo 3

n . pk

Demostracin: El nmero de factores contribuyendo un factor de p es n/p , el nmero de factores contribuyendo un segundo factor de p es n/p2 , etc..u
121 Teorema Si a, b son relativamente primos, entonces
a 1

k=1

kb = a

b1

k=1

ka (a 1)(b 1) = . b 2

Demostracin: Considrese el rectngulo con vrtices en (0, 0), (0, b), (a, 0), (a, b). Este rectngulo contiene (a 1)(b 1) puntos cuyas coordenadas son ambas enteras. El rectngulo se divide en dos partes gracias a la xb recta y = . Se demostrar que no hay puntos de coordenadas ambas enteras sobre esta recta, a excepcin de los a n b extremos. Si hubiese puntos de coordenadas enteras (m, n), 0 < m < a, 0 < n < b, entonces = . Luego n/m m a kb es una fraccin reducida de la fraccin irreducible b/a, contradiccin. Los puntos Lk = (k, ), 1 k a 1 a kb estn sobre esta recta. Ahora bien, es el nmero de puntos de coordenadas enteras sobre la recta vertical a a 1 kb kb que va desde (k, 0) hasta (k, ), esto es, , que es el nmero de puntos con coordenadas enteras en la a a
k=1

ka cuenta el nmero de puntos con coordenadas enteras b k=1 en la parte superior del rectngulo. Como hay (a 1)(b 1) puntos de coordenadas enteras en total, y su nmero es compartido de manera igual por ambas partes, queda demostrado el teorema. u parte inferior del rectngulo. De igual manera,
122 Denicin Llmase espectro de un nmero real a la sucesin innita

b1

Esp () = { , 2 , 3 , . . .}. Dos sucesiones Esp () y Esp () se dicen complementarias si hacen una particin de los nmeros naturales no nulos, esto es, Esp () Esp () = y Esp () Esp () = N. Demostrarase ms adelante que las sucesiones Esp 2 = {1, 2, 4, 5, 7, 8, 9, 11, 12, 14, 15, 16, 18, 19, 21, 22, 24, 25, . . .}, y son complementarias.
123 Teorema (Teorema de Beatty, 1926) Si > 1 es irracional y

Esp 2 + 2 = {3, 6, 10, 13, 17, 20, 23, 27, 30, 34, 37, 40, 44, 47, 51, . . .}

1 1 + = 1, entonces las sucesiones Esp () y Esp () son complementarias.

La funcin parte entera Demostracin: Ya que > 1, > 1, Esp () y Esp () son ambas sucesiones de trminos distintos y el total de trminos no excediendo N en una y otra sucesin es N/ + N/ . Pero N/ 1 + N/ 1 < N/ + N/ < N/ + N/, siendo la ltima desigualdad estricta ya que ambos , son irracionales. Como 1/ + 1/ = 1, se deduce que N 2 < N/ + N/ < N. Como la cantidad emparedada es entera, se deduce que N/ + N/ = N 1. As, la cantidad total de trminos que no exceden a N en Esp () y Esp () es N 1. Como esto es cierto para cada N 1, cada intervalo (n, n + 1) contiene exactamente uno de los trminos de Esp () y Esp (). Se sigue que Esp () Esp () = N, Esp () Esp () = . u

25

Se observa tambin un resultado en la direccin opuesta.


124 Teorema (Teorema de Bang, 1957) Si las sucesiones

Spec() y Spec() son complementarias, entonces , son nmeros irracionales positivos que satisfacen 1 1 + = 1. Demostracin: Si ambos , fueren nmeros racionales, entonces eventualmente Spec(), Spec() contendran los mismos enteros, y por lo tanto no seran disjuntas. As pues, y son irracionales. Si 0 < 1, dado n entonces existe m para el cual m 1 < n m; luego n = m , lo que implica que Spec() = N, de donde > 1 (y con esto tambin > 1). Si la interseccin Spec() Spec() fuese nita, entonces
n

lm

n/ + n/ = 1, n

pero entonces ( n/ + n/ )

125 Ejemplo Hllese un polinomio no idnticamente nulo P(x, y) tal que para todo nmero real t se tenga P( 2t , 3t ) = 0.

1 1/ + 1/ ya que n , se colige que 1/ + 1/ = 1. u n

Resolucin: Comprobarase que 3 2t 2 3t = 0, 1 o 2. Luego entonces se podr elegir P(x, y) = (3x 2y)(3x 2y 1)(3x 2y + 1)(3x 2y + 2).

Para vericar la asercin, obsrvese que como x tiene perodo unitario, es suciente comprobar la asercin para t [0; 1[. Divdase [0; 1[ as [0; 1[= [0; 1/3[[1/3; 1/2[[1/2; 2/3[[2/3; 1[. Si t [0; 1/3[, entonces tanto 2t como 3t son = 0, y as 3 2t 2 3t = 0. Si t [1/3; 1/2[ entonces 3t = 1 y 2t = 0, y as 3 2t 2 3t = 2. Si t [1/2; 2/3[, luego 2t = 1, 3t = 1, dando 3 2t 2 3t = 1. Si t [2/3; 1[, se tendr 2t = 1, 3t = 2, y 3 2t 2 3t = 1.
126 Ejemplo Descrbase todos los enteros positivos n para los cuales 1 +

2n

2n.

26

Captulo 3

Resolucin: Sea 2n = m(1 + 2n Si m ). 2n 1 entonces 2n ( 2n 1)( 2n + 1) = 2n 2 1 2n + 1, entonces 2n ( 2n 2 + 1)2 2n + 1, otra contradiccin. Por lo tanto 2n 1 < 2n, contradiccin. Si m se debe tener m = 2n . l(l + 1) 2n . Luego todos los enteros con la propiedad Recprocamente, sea n = . Ya que l < 2n < l + 1, se tiene l = 2 deseada son nmeros triangulares.
127 Ejemplo Demustrese que la sucesin de enteros

1+ 2 donde n es un entero no negativo, es alternadamente par e impar. Resolucin: En virtud del teorema del binomio, (1 + 2)n + (1 2)n = 2

(2)k
0kn/2

n := 2N, 2k

un nmero par. Como 1 < 1 2 < 0, entonces (1 2)n es la parte fraccionaria de (1 + 2)n o de (1 + 2)n + 1 dependiendo de la paridad de n. Para n impar, (1 + 2)n 1 < (1 + 2)n + (1 2)n < (1 + 2)n , de donde (1 + 2)n + (1 n = (1 + 2)n , que es siempre par, y para n par 2N := (1 + 2)n + (1 2)n = 2) n (1 + 2) + 1, y as (1 + 2)n = 2N 1, es siempre impar para n par.
128 Ejemplo Demustrese que los primeros mil dgitos de

(6 + 35)1980 luego del punto decimal son todos 9s. (6 + 35)1980 + (6 35)1980 = 2k, 1 es un entero par. Ahora bien, 0 < 6 35 < 1/10 (si no < 6 35, y al cuadrar 3500 < 3481, contradiccin) y luego 10 0 < (6 35)1980 < 101980 . Se deduce que 2k 1 + 0,9 . . . 9 = 2k
1979 nines

Resolucin: Gracias al ejemplo 127,

1 101980

< (6 + 35)1980 < 2k,

demostrando la asercin.
129 Ejemplo (Putnam 1948) Si n es un entero positivo, demustrese que

n+ n+1 = 4n + 2 . Resolucin: Elevando al cuadrado se ve que 4n + 1 < n + n + 1 < 4n + 3. Ni 4n + 2 ni 4n + 3 son cuadrados, ya que todo cuadrado es congruente a 0 o 1 mdulo 4, luego 4n + 2 = 4n + 3 , de donde se colige el resultado.

La funcin parte entera


130 Ejemplo Hllese una frmula para el ensimo entero positivo no cuadrado.

27

Resolucin: Sea Tn el ensimo entero positivo no cuadrado. Luego, existe un nmero entero positivo m tal que m2 < Tn < (m + 1)2 . Ya que hay m cuadrados menores que Tn se tiene que Tn = n + m. Entonces se ve que m2 < n + m < (m + 1)2 o sea m2 m < n < m2 + m + 1. Como n, m2 m, m2 + m + 1 son todos enteros, las desigualdades anteriores implican que m2 m + 1 1 < n < m2 + m + , 4 4 1 n+ . Luego el ensimo entero positivo no cuadrado es 2

esto es, (m 1/2)2 < n < (m + 1/2)2 . Pero entonces m = Tn = n + n + 1/2 .


131 Ejemplo (Putnam 1983) Sea f(n) = n +

n . Demustrese que para cada entero positivo m, la sucesin

m, f(m), f(f(m)), f(f(f(m))), . . . tiene al menos el cuadrado de un entero. Resolucin: Sea m = k2 + j, 0 j 2k. Divdase las ms en dos grupos: aqullas (grupo A) para las que j, 0 j k y stas (grupo B) para las que j, k < j < 2k + 1. Obsrvese que k2 m < (k + 1)2 = k2 + 2k + 1. Si j = 0, no hay nada que demostrar. Supngase primero que m B. Como m = k, se tiene f(m) = k2 + j + k = (k + 1)2 + j k 1, con 0 j k 1 k 1 < k + 1. Esto quiere decir que f(m) o bien es un cuadrado, o bien f(m) A. As pues, es slo necesario considerar la alternativa m A, en cuyo case m+k = k y f(f(m)) = f(m + k) = m + 2k = (k + 1)2 + j 1. Esto signica que o bien f(f(m)) es un cuadrado, o bien f(f(m)) A con un exceso j 1 menor que el exceso j de m. Luego de cada iteracin el exceso se reduce, y eventualmente ser cero, en cuyo caso se obtendr un cuadrado.
132 Ejemplo Resulvase la ecuacin

x2 x 2 = x , para x R. Resolucin: Obsrvese que a = b si y slo si k Z with a, b [k, k + 1), lo que sucede si y slo si a b < 1. Luego, la ecuacin dada tendr solucin si y solamente si x2 2x 2 < 1, de donde el conjunto solucin es x R : x ] 1 :
133 Ejemplo Encuntrese la parte entera

1 1 1 (1 5)] [ (1 + 17), (1 + 21)[ . 2 2 2

1 . k k=1 Resolucin: La funcin x x1/2 es decreciente. Luego, para todo entero positivo k, 1 < k+1
k +1 k

106

dx 1 < . x k

28 Sumando de k = 1 hasta k = 106 1 se deduce que 1 < k k=2 Se verica fcilmente que la integral es 1998. Luego 1998 + 1/10 < La parte entera es as 1998.
134 Ejemplo En cuntos ceros termina 300!?
3 106 106 1

Captulo 3

dx < x

106 1

k=1

1 . k

1 < 1999. k k=1

106

Resolucin: El nmero de ceros queda determinado por la potencia mayor de 10 que divida a 300!. Ya que abundan ms los mltiplos de 2 en 300! que los mltiplos de 5, el nmero de ceros queda determinado por la potencia mayor de 5 que divida a 300!. En virtud de la frmula de De Polignac, la buscada potencia es
k=1

300/5k = 60 + 12 + 2 = 74.

135 Ejemplo Divide 7 a

1000 ? 500

Resolucin: La potencia mayor de 7 que divide a 1000! is 1000/7 + 1000/72 + 1000/73 = 142 + 20 + 2 = 164, gracias a la frmula de De Polignac (teorema 120). De manera semejante, la potencia mayor de 7 que divide a 500! is 1000 1000 1000! 71 + 10 + 1 = 82. Ya que = , la potencia mayor de 7 que divide a es 164 2 82 = 0, de donde 2 500 (500!) 500 1000 se colige que el 7 no divide a . 500
136 Ejemplo Sea n = n1 + n2 + + nk donde los ni son enteros no negativos. Demustrese que la cantidad

n! n1 !n2 ! nk ! es entera. Resolucin: Por 3 del teorema 119 se deduce mediante induccin que a1 + a2 + + al a1 + a2 + + al . Por la frmula de De Polignac (teorema 120) la potencia de un primo p que divide a n! n/pj =
j1 j1

(n1 + n2 + + nk )/pj .

Luego la potencia de p que divide a n1 !n2 ! nk ! es


j1

n1 /pj + n2 /pj + nk /pj .

Ya que n1 /pj + n2 /pj + + + nk /pj (n1 + n2 + + nk )/pj ,

Tarea se colige que la potencia de cualquier primo que divida al numerador de n! n1 !n2 ! nk ! es al menos tan grande como la potencia del mismo primo que divida al denominador. Esto demuestra la asercin.

29

137 Ejemplo Dado un entero n > 3, demustrese que el mnimo comn mltiplo de los productos, x1 x2 xk (k 1), cuyos factores xi son enteros positivos satisfaciendo x1 + x2 + xk n,

es menor que n!. Resolucin: Se demostrar que el mnimo comn mltiplo en cuestin es p
p

n/p

p primo

Considrese un producto arbitrario x1 x2 xk , y un primo arbitrario p. Supngase que pj divide a xj pero que pj +1 no divide a xj . Claramente, p1 + + pk n y como p p, se tiene p(1 + k ) n o 1 + + k n . p

Luego, se sigue que el exponente de un primo arbitrario dividiendo el mnimo comn mltiplo p es a lo sumo p/n . Pero en tomando x1 = = xk = p, k = n/p , se observa que para al menos un producto se consigue la igualdad. Esto demuestra la asercin.
138 Ejemplo Supngase que se criba los enteros positivos de la manera siguiente: se toma a1 = 1 y se deja a1 + 1 = 2. El

prximo trmino es 3, al que se llamar a2 y tomar, y luego se dejar a2 + 2 = 5. El prximo entero disponible es 4 = a3 , y luego se dejar a3 + 3 = 7, etc. As se tomar los enteros 1, 3, 4, 6, 8, 9, 11, 12, 14, 16, 17, . . . . Encuntrese una frmula para an . Resolucin: Se pide una sucesin {Sn } complementaria a {Sn + n}. Por el teorema de Beatty (teorema 123), n y n + n = n( + 1) son complementarias si 1/ + 1/( + 1) = 1. Pero luego = (1 + 5)/2, la razn dorada. El ensimo trmino es pues an = n .

Tarea

Captulo

Congruencias
4.1. Congruencias
139 Denicin (Congruencias) Sean n 1, a, b enteros. Dcese que a b md n (lase a es congruente a b mdulo o n) si a y b dejan el mismo residuo al ser divididos por n, o, equivalentemente, si n(a b). 140 Teorema (Propiedades de las congruencias) Sean n 1, a, b, c, d enteros. Si a b m d n y si c d m d n o o

entonces

a + c b + d m d n, o ac bd m d n. o

Demostracin: Se tiene n(a b) y n(c d). Luego hay enteros u, v con nu = a b y nv = c d. As (a + c) (b + d) = n(u + v) = = n((a + c) (b + d)) a + c b + d m d n, o

y adems ac bd = a(d + nv) d(a nu) = n(av + du) de donde se obtiene el teorema. u
141 Corolario Si j 1 es entero y si a b m d n entonces aj bj m d n. o o

= =

n(ac bd) ac bd m d n, o

Demostracin: Basta utilizar induccin en el segundo inciso del Teorema 140, poniendo c = aj1 y d = bj1 . u
142 Corolario Si

p(x) = 0 + 1 x + + n xn es polinomio cuyos coecientes son todos enteros y si a b m d n entonces p(a) p(b) m d n. o o Demostracin: Utilcese el Corolario 141 para demostrar que i ai i bi m d n y smese cada trmino. u o 30

Congruencias

31

143 Teorema Sean a, b, n enteros. Si la congruencia ax b m d n posee al menos una solucin, entonces tiene exactao mente (a, n) soluciones incongruentes m d n. o

Demostracin: Del teorema 71, todas las soluciones de la ecuacin diofntica ax + ny = b son de la forma x = x0 + nt/d, y = y0 at/d, d = (a, n), t Z, en donde x0 , y0 satisfacen ax0 + ny = b. Dejando que t corra por los valores t = 0, 1, . . . ((a, n) 1), se obtiene (a, n) soluciones que son mutuamente incongruentes, ya que el valor absoluto de la differencia de cualesquiera dos de ellas es menor que n. Si x = x0 + nt /d es cualquier otra solucin, se escribe t como t = qd + r, 0 r < d. Entonces x = x0 + n(qd + r)/d = x0 + nq + nr/d x0 + nr/d m d n. o

Luego, toda solucin de la congruencia ax b m d n es congruente m d n a uno y solamente uno de los o o d valores x0 + nt/d, 0 t d 1. Quirese decir que si existiese una solucin de la congruencia, entonces existirn d soluciones incongruentes m d n.u o
144 Teorema Sean x, y enteros y sean a, n enteros diferentes de cero. Entonces

ax ay si y slo si xy m d o

m d n o n . (a, n)

Demostracin: Si ax ay m d n entonces a(x y) = sn para algn entero s. Esto implica que o (x y) Ya que (a/(a, n), n/(a, n)) = 1, se deduce que n |(x y), (a, n) gracias al lema de Euclides (lema 56). Esto implica que xy Recprocamente, si se tiene x y m d o m d o n . (a, n) n a =s . (a, n) (a, n)

n entonces se tendr (a, n) ax ay m d o an , (a, n)

en multiplicando por a. Como (a, n) divide a a, la congruencia anterior implica, a fortiori que ax ay = tn para algn entero t. Esto termina la demostracin.u
145 Corolario Si ax ay m d n y (a, n) = 1, entonces x y m d n. o o 146 Ejemplo Demustrese que 7 divide a 22225555 + 55552222 utilizando congruencias. Esta pregunta se vi ya en el

problema 39. Resolucin: 2222 3 mod 7, 5555 4 m d 7 y 35 5 m d 7. Ahora bien, 22225555 + 55552222 35555 + 42222 o o (35 )1111 + (42 )1111 51111 51111 0 m d 7, lo que demuestra la asercin. o

32
147 Ejemplo Hallse el residuo cuando 61987 es dividido por 37.

Captulo 4

Resolucin: 62 1 m d 37. As pues, 61987 6 61986 6(62 )993 6(1)993 6 31 m d 37. o o


148 Ejemplo Encuntrese todas las soluciones de 5x 3 m d 7 o

Resolucin: De acuerdo al teorema 143 existe una solucin nica a la congruencia m d 7 por ser (5, 7) = 1. Gracias al o algoritmo de Euclides, 7 = 51+2 5 = 22+1 2 = 2 1. As, 1 2 dando 1 = 5 2 2 = 5 2(7 5 1) = 5 3 7 2. De aqu, 3 = 5(9) 7(6). De sto resulta que 5 9 3 m d 7, o sea, 5 2 3 m d 7. As pues x 2 m d 7. o o o
149 Ejemplo Resulvase la congruencia

= 522 = 7 5 1,

3x 6

m d 12. o

Resolucin: Como (3, 12) = 3 y 3|6, la congruencia tiene tres soluciones incogruentes. Por inspeccin x = 2 es na solucin. En virtud del teorema 71, todas las soluciones son de la forma x = 2 + 4t, t Z. Poniendo t = 0, 1, 2, se obtienen las tres soluciones incongruentes t = 2, 6, 10 mdulo 12.
150 Ejemplo (USAMO, 1979) Determnese todas las soluciones no negativas

(n1 , n2 , . . . , n14 ) de la ecuacin diofntica n4 + n4 + + n4 = 1599 1 2 14 de haberlas. Resolucin: No hay tales soluciones. Todas las cuartas potencias m d 16 son o bien 0 o bien 1 m d 16. Esto signica o o que n4 + + n4 1 14 es a lo sumo 14 m d 16. Pero 1599 15 m d 16. o o

Tarea
151 Problema Hallse el ltimo dgito de 3100 . 152 Problema Hallse el dgito de las unidades de 77 . 153 Problema Comprubese que 7 divide a 32n+1 + 2n+2 para todo nmero entero n > 0. 154 Problema (Olimpada polaca) Qu dgitos debe substituirse por a y b en 30a0b03 de tal manera que el entero resul7

tante sea divisible por 13 ?

Sistemas residuales completos y reducidos


155 Problema Comprubese que la ecuacin x2 5y2 = 2 no tiene soluciones enteras. 156 Problema Comprubese la siguiente observacin de Euler: 232 + 1 es divisible por 641. 157 Problema Hallse un nmero innito de enteros n tal que 2n + 27 sea divisible por 7. 158 Problema Existe acaso enteros positivos x, y tal que x3 = 2y + 15? 159 Problema Comprubese que 2k 5, k = 0, 1, 2, . . . nunca deja residuo 1 cuando es dividido por 7.

33

4.2. Sistemas residuales completos y reducidos


160 Denicin Si a b m d n entonces b es llamado un residuo de a mdulo n. Un conjunto o

{a1 , a2 , . . . an } es llamado un sistema completo de residuos mdulo n si para cada entero b existe exactamente uno ndice j para el cual b aj m d n. En particular, al conjunto {0, 1, . . . , n 1} se le llama conjunto cannico de residuos mdulo n. o
161 Denicin Sea n > 1. Los (n) enteros

1 = a1 < a2 < < a(n) = n 1 menores que n y relativamente primos a n reciben el nombre de residuos cannicos reducidos mdulo n.
162 Denicin Sea n > 1 un entero. Un entero b es llamado inverso multiplicativo de un entero a mdulo n si ab 1

m d n. o

163 Teorema Si existiese, el inverso multiplicativo x de un entero a mdulo el entero n > 1, es nico.

Demostracin: Si x, y fuesen inversos de a m d n entonces ax 1 m d n y tambin ay 1 m d n. o o o Multiplicando por y la primera congruencia, se ve que (ya)x y m d n. As, x y m d n. u o o
164 Teorema Sean n > 1, a enteros. Entonces a posee un inverso mdulo n si y slo si a es relativamente primo a n.

Demostracin: Presmase que b es un inverso de a m d n. Luego ab 1 m d n que conlleva la existencia o o de un entero s tal que ab 1 = sn, esto es, ab sn = 1. sta es una combinacin lineal de a y n, luego ser divisible por (a, n), dando (a, n) = 1. Recprocamente, si (a, n) = 1, por el teorema de Bachet-Bezout existe enteros x, y tales que ax + ny = 1. Esto da de inmediato ax 1 m d n, lo cual quiere decir que a tiene como inverso a x, m d n. o o

165 Corolario Si n > 1 es entero, entonces existe solamente (n) enteros invertibles, mdulo n. 166 Teorema Si a es relativamente primo al entero positivo integer n, entonces existe un entero positivo k n tal que ak 1

m d n. o

Demostracin: Ya que (a, n) = 1 se tiene (aj , n) = 1 para toda j 1. Considrese la sucesin a, a2 , a3 , . . . , an+1 m d n. Como sta posee n + 1 trminos y hay slo n residuos diferentes mdulo n, el principio de las pichoneras o

34

Captulo 4 de Dirichlet implica que dos de estas potencias estn en la misma clase residual m d n. sto es, se puede hallar o s, t con 1 s < t n + 1 tal que as at m d n. Ahora bien, 1 t s n. Luego as at m d n resulta o o en ats as ats at m d n, lo que quiere decir que at ats at m d n. Utilizando el corolario 145 se o o deduce que ats 1 m d n, lo que demuestra el resultado.u o

Si (a, n) = 1, el teorema anterior dice que existe un entero positivo k con ak 1 m d n. Por el axioma del buen orden, o existe un entero mnimo satisfaciendo esta congruencia, lo que sugiere la prxima denicin.
167 Denicin Si m es el entero positivo mnimo con am 1 m d n, entonces se side que a tiene orden m m d n. o o 168 Teorema Sea n > 1 un entero. Entonces a Z tiene un orden m d n si y solamente si (a, n) = 1. o

Demostracin: Si (a, n) = 1, entonces a tiene un orden en virtud del teorema 166 y el axioma del buen orden. Presmase pues que a has an order m d n. Evidentemente a = 0. La existencia de un orden conlleva la o existencia de un entero positivo m tal que am 1 m d n. Luego, existe un entero s con am + sn = 1, o sea, o a am1 + sn = 1. Esta es una combinacin lineal de a y de n por lo cual es divisible por (a, n). De aqu se deduce que (a, n) = 1. u
169 Teorema Sea (a, n) = 1 y sea t un entero. Entonces at 1 m d n si y slo si ordn a|t. o

Demostracin: Presmase que ordn a|t. Luego existe un entero s con sordn a = t, de donde se deduce at asordn a (aordn a )s 1s 1 m d n. o

Recprocamente, presmase que at 1 m d n y que t = x ordn a + y, 0 y < ordn a. Entonces o ay atxordn a at (aordn a )x 1 1x 1 m d n. o

Si y > 0 entonces se tendra un entero positivo menor que ordn a poseyendo la propiedad ay 1 m d n, lo que o contradice la denicin de ordn a como el menor entero positivo con esta propiedad. As pues y = 0 y entonces t = x ordn a, esto es, ordn a|t. u

170 Teorema Sea n > 1, a Z, (a, n) = 1. Si r1 , r2 , . . . , r(n) es un sistema de residuos reducidos mdulo n, entonces

tambin ar1 , ar2 , . . . , ar(n) es un sistema de residuos reducidos mdulo n.

Demostracin: Se necesita demostrar que los (n) enteros ar1 , ar2 , . . . , ar(n) son mutuamente incongruentes m d n. Supngase que ari arj m d n para algn i = j. Como (a, n) = 1, se deduce del corolario o o 145 que ri rj m d n. sto contradice el hecho que las rs son incongruentes, con lo que queda demostrado el o teorema.u
171 Corolario Sea n > 1, a, b Z, (a, n) = 1. Si r1 , r2 , . . . , r(n) es un sistema de residuos reducidos mdulo n, entonces

ar1 + b, ar2 + b, . . . , ar(n) + b es tambin un sistema de residuos reducido mdulo n.

172 Ejemplo Encuntrese el inverso de 5 m d 7. o

Resolucin: Bscase una solucin a 5x 1 m d 7. Por inspeccin se ve que la solucin buscada es x 3 m d 7. o o


173 Ejemplo Hllese el orden de 8 m d 11. o

Tarea

35

Resolucin: Por el corolario ?? ord11 8 es uno de entre 1, 2, 5 o 10. Ahora 82 2 m d 11, 84 4 m d 11 y 85 1 o o m d 11. El orden es pues ord11 8 = 10. o
174 Ejemplo (Putnam, 1956) Demustrese que todo entero positivo posee un mltiplo cuya expansin decimal involucra

todos los 10 dgitos. Resolucin: Se n un entero positivo arbitrario con k dgitos, y sea m = 123456780 10k+1. Entonces los n consecutivos m + 1, m + 2, . . . m + n comienzan en 1234567890, forman un sistema completo de residuos mdulo n y luego uno de ellos es divisible por n.

Tarea 4.3. Teoremas de Fermat, Wilson y Euler


175 Teorema (Pequeo teorema de Fermat) Sea p un primo y a un entero tal que p |a. Entonces

ap1 1

m d p. o

Demostracin: Ya que (a, p) = 1, los enteros a 1, a 2, . . . , a (p 1) tambin forman un sistema de residuos reducido m d p en vista del corolario 171. Luego pues o (a 1)(a 2) (a (p 1)) 1 2 (p 1) o sea, Como ((p 1)!, p) = 1, se puede cancelar las (p 1)!s gracias al corolario 145. u
176 Corolario Para todo primo p y para todo entero a,

m d p, o

ap1 (p 1)! (p 1)!

m d p. o

ap a

m d p. o

Demostracin: O bien p|a o p |a. Si p|a, entonces a 0 ap m d p y no hay nada que demostrar. Si p |a, o el pequeo teorema de Fermat dice que p|ap1 1. As pues p|a(ap1 1) = ap a, que da el resultado.u
177 Corolario Sea p un primo y a un entero. Si p |a entonces ordp a|p 1.

Demostracin: El resultado se consigue de inmediato por el teorema 169 y el pequeo teorema de Fermat.u
178 Lema Si a2 1 m d p, o bien a 1 m d p o bien a 1 m d p. o o o

Demostracin: Se tiene p|a2 1 = (a 1)(a + 1). Como p es primo, p debe dividir a al menos uno de estos dos factores, lo que demuestra el lema.u
179 Teorema (Teorema de Wilson) Si p es primo, entonces (p 1)! 1 m d p. o

Demostracin: Si p = 2 o p = 3, el resultado se deduce por vericacin directa. Presmase que p > 3. Considrese a, 2 a p 2. A cada a se le asocia su inverso nico a m d p, i.e. aa 1 m d p. Observse que o o a = a porque si no, a2 1 m d p violando el lema anterior, ya que a = 1, a = p 1. En nultiplicando las as o con 2 a p 2, se aparean stas con sus inversos, y la contribucin neta de este producto es por lo tanto 1. En smbolos, 2 3 (p 2) 1 m d p. o

36 En otras palabras,

Captulo 4

(p 1)! 1 Esto conlleva al resultado. u

j
2 a p 2

(p 1) 1 1 (p 1) 1

m d p. o

180 Teorema (Teorema de Euler) Si (a, n) = 1, entonces a(n) 1 m d n. o

Demostracin: Sean a1 , a2 , . . . , a(n) los residuos cannicos reducidos m d n. Como (a, n) = 1, aa1 , aa2 , . . . , aa(n) , o tambin forman un sistema de residuos reducidos, gracias al corolario 171. As pues, aa1 aa2 aa(n) a1 a2 a(n) o a(n) a1 a2 a(n) a1 a2 a(n) m d n. o m d n, o

Como (a1 a2 a(n) , n) = 1, se puede cancelar el producto a1 a2 a(n) en uno y otro lado, de donde se deduce el teorema. u
181 Corolario Si (a, n) = 1, entonces ordn a|(n). 182 Ejemplo Sea a1 = 4, an = 4an1 , n > 1. Hllese el residuo cuando a100 se divide por 7.

Resolucin: Por el pequeo teorema de Fermat, 46 1 m d 7. Como 4n 4 m d 6 para todos los enteros positivos n, se o o tiene 4n = 4 + 6t para algn entero t. As a100 4a99 44+6t 44 (46 )t 4 m d 7. o

183 Ejemplo Demustrese que m, n Z, mn(m60 n60 ) es siempre divisible por 56786730.

Resolucin: Obsrvese que a = 56786730 = 2 3 5 7 11 13 31 61. Sea Q(x, y) = xy(x60 y60 ). Obsrvese que (x y)|Q(x, y), (x2 y2 )|Q(x, y), (x3 y3 )|Q(x, y), (x4 y4 )|Q(x, y), (x6 y6 )|Q(x, y), (x10 y10 )|Q(x, y), (x12 y12 )|Q(x, y), y (x30 y30 )|Q(x, y). Si p es cualquiera de los primos dividiendo a a, se tiene entonces que mp m 0 m d p y que np n 0 m d p. o o As n(mp m) m(np n) 0 m d p, sto es, mn(mp1 np1 ) 0 m d p. Luego se tiene que o o 2|mn(m n)|Q(m, n), 3|mn(m2 n2 )|Q(m, n), 5|mn(m4 n4 )|Q(m, n), 7|mn(m6 n6 )|Q(m, n), 11|mn(m10 n10)|Q(m, n), 13|mn(m12 n12 )|Q(m, n), 31|mn(m30 n30 )|Q(m, n) y 61|mn(m60 n60 )|Q(m, n). Como todos estos son primos distintos, se deduce que a|mnQ(m, n), como se quera demostrar.
184 Ejemplo Si p 1 m d 4, demustrese que o

p1 ! 1 2

m d p. o

Tarea

37

Resolucin: En el producto (p 1)!, se aparea j, 1 j (p 1)/2 con p j. Obsrvese que j(p j) j2 m d p. o Luego p1 1 (p 1)! j2 (1)(p1)/2 ! m d p. o 2
1j(p1)/2

El resultado se consigue al observar que (1)(p1)/2 = 1.


185 Ejemplo (IMO 1970) Demustrese que para todo entero n es imposible partir el conjunto

{n, n + 1, n + 2, n + 3, n + 4, n + 5} en dos subconjuntos tales que el producto de los miembros del uno sea igual al producto de los miembros del otro. Resolucin: Supngase en miras de contradiccin que existiese tal n con tal particin, el primer subconjunto teniendo producto de miembros A y el otro teniendo producto de miembros B. O bien, uno de los enteros en {n, n+1, n +2, n +3, n +4, n +5} es divisible por 7, en cuyo caso exactamente uno de entre A o B es divisible por 7, y por lo tanto, A B no es divisible por 72 y as A B no es un cuadrado. En la segunda posibilidad, todos los miembros del conjunto son relativamente primos a 7. Esto quiere decir que, gracias al teorema de Wilson, n(n + 1) (n + 6) 1 2 6 A B 1 m d 7. o

Pero si A = B entonces A2 1 m d 7, lo que es imposible ya que 1 no es cuadrado m d 7. o o

Tarea
186 Problema Sea n N, n 2. Demustrese que si N est la suma de n nmeros impares consecutivos, entonces N no

puede ser primo.

4.4. Teorema snico de los residuos


187 Teorema (Teorema snico de los residuos o Teorema de Sun Tsu) Sean m1 , m2 , . . . mk enteros relativamente primos por pares, todos mayores que 1, y sean a1 , a2 , . . . ak enteros arbitrarios. Luego el sistema de congruencias

x x . . . x posee una solucin nica mdulo m1 m2 mk .

. . .

a1 a2 . . . ak

m d m1 o m d m2 o m d mk o

m1 m2 mk , 1 j k y sea Qj el inverso de Pj m d mj , i.e., Pj Qj 1 o mj m d mj , que sabemos que existe, ya que las mi son relativamente primas por pares. Pngase o Demostracin: Pngase Pj = x = a1 P1 Q1 + a2 P2 Q2 + + ak Pk Qk . Este nmero claramente satisface las condiciones descritas en el teorema. La unicidad de esta solucin es fcil de establecer mdulo m1 m2 mk . u
188 Ejemplo Puede encontrarse un milln de enteros positivos que sean divisibles por al menos un cuadrad?

38

Captulo 4

Resolucin: La respuesta es armativa. Sean p1 , p2 , . . . , p1000000 un milln de primos diferentes. Por el teorema snico de los residuos existe una solucin del siguiente sistema de congruencias: x x . . . x . . . 1 2 . . . m d p2 , o 1 m d p2 , o 2 m d p2 o 1000000 .

. . .

1000000

Los nmeros x + 1, x + 2, . . . , x + 1000000 son un milln de enteros consecutivos, cada uno divisible por el cuadrado de un primo.

4.5. Criterios de divisibilidad


189 Teorema Un entero n es divisible por 5 si y solamente si su ltimo dgito es o un 0 o un 5.

Demostracin: Se derivar el resultado para n > 0, ya que para n < 0 slo basta aplicar el teorema a n > 0. Como 10k 0 m d 5 para enteros k 1, se tiene o n = as 10s + as1 10s1 + + a1 10 + a0 a0 m d 5, o

As pues, la divisibilidad de n por 5 depende en si a0 es divisible por 5, lo que slo pasa cuando a0 = 0 o a0 = 5. u
190 Teorema Sea k un entero positivo. Un entero n es divisible por 2k si y solamente si el nmero formado por los ltimos

k dgitos de n es divisible por 2k . Demostracin: Si n = 0, no hay nada que demostrar. Si se demuestra el resultado para n > 0 entonces el resultado para n < 0 se deduce al aplicar lo ya obtenido a n = (1)n > 0. Presmase pues que n Z, n > 0 y sea su expansin decimal n = as 10s + as1 10s1 + + a1 10 + a0 , donde 0 ai 9, as = 0. Ahora bien, 10t = 2t 5t 0 m d 2t para t k. Luego pues, o n = as 10s + as1 10s1 + + a1 10 + a0 ak1 10k1 + ak2 10k2 + + a1 10 + a0

m d 2k , o

de donde n es divisible por 2k si y solamente si el nmero formado por los ltimos k dgitos de n es divisible por 2k . u
191 Teorema (Regla de los 9s) Un entero n es divisible por 9 si y solamente si la suma de sus dgitos es divisible por 9.

Demostracin: Si n = 0 no hay nada que demostrar. Si se demuestra el resultado para n > 0 entonces se puede deducir el resultado para n < 0 en aplicando lo ya obtenido al nmero n = (1)n > 0. Presmase pues que n Z, n > 0 y que su expansin decimal es n = as 10s + as1 10s1 + + a1 10 + a0 , donde 0 ai 9, as = 0. Observse que 10 1 m d 9 y que 10t 1t 1 m d 9. Ahora bien, o o n = de donde se colige el resultado. u as 10s + as1 10s1 + + a1 10 + a0 as + + a1 + a0 m d 9, o

Tarea

39

 Como 10 1
es divisible por 3.

m d 3 se puede tambin ver que n es divisible por 3 si y solamente si la suma de su dgitos o

192 Denicin Si el entero n tiene expansin decimal

n = as 10s + as1 10s1 + + a1 10 + a0 , entonces su suma alternante de dgitos es as as1 + as2 as3 + + (1)s1 a0
193 Teorema Un entero n es divisible por 11 si y solamente si su suma de dgitos alternante es divisible por 11.

Demostracin: Presmase que n > 0. Sea n = as 10s + as1 10s1 + + a1 10 + a0 , donde 0 ai 9, as = 0. Observse que 10 1 m d 11y as 10t (1) m d 11. Luego o o n = as 10s + as1 10s1 + + a1 10 + a0 as (1)s + as1 (1)s1 + as2 (1)s2 + + a1 + a0 m d 11 o

y se colige el resultado. u

Tarea
194 Ejemplo Para cuntos enteros n en {1, 2, 3, . . . , 100} es el dgito de las decenas de n2 impar?

Resolucin: En el subconjunto {1, 2, . . . 10} hay slo dos valores de n (4 y 6) para los cuales el dgito de las decenas de n2 es impar. Ahora bien, (n + 10)2 = n2 + 20n + 100 tiene la misma paridad en su dgito de las decenas que el dgito de las decenas de n2 . Luego, hay 2 10 = 20 enteros n para los cuales se verica la condicin prescrita.
195 Ejemplo Hallar todos los enteros con dgito inicial 6 tales que si se les suprime este dgito incial,el nmero resultante es

1/25 del nmero original. Resolucin: Sea x el entero buscado. Entonces x = 6 10n + y donde y es un entero positivo. La condicin del problema estipula que 1 (6 10n + y) , y= 25 o sea, 10n y= = 25 10n2. 4 Esto requiere n 2 y por lo tanto y = 25, 250, 2500, 25000, etc.. Luego x = 625, 6250, 62500, 625000, etc..
196 Ejemplo Sea A un entero positivo y A sea el entero positivo resultante de alguna permutacin especca de los dgitos

de A. Demustrese que si A + A = 1010 entonces A es divisible por 10. Resolucin: Claramente, A y A debern tener 10 dgitos cada uno. Pongas pues A = a10 a9 a8 . . . a1 y A = b10 b9 b8 . . . b1 ,

40

Captulo 4

donde ak , bk , k = 1, 2, . . . , 10 son los dgitos de A y A respectivamente. Ahora, como A + A = 10000000000, deberemos tener que a1 + b1 = a2 + b2 = = ai + bi = 0 y ai+1 + bi+1 = 10, ai+2 + bi+2 = = a10 + b10 = 9, para algn subndice i, 0 i 9. Note que si i = 9 no hay ninguna suma de las ai+2 + bi+2 , ai+3 + bi+3 , . . . y si i = 0 no hay ninguna suma de las a1 + b1 , . . . , ai + bi . Sumando, a1 + b1 + a2 + b2 + + ai + bi + ai+1 + bi+1 + + a10 + b10 = 10 + 9(9 i). Ahora bien, si i es par, 10 + 9(9 i) es impar y si i es impar 10 + 9(9 i) es par. Pero como a1 + a2 + + a10 = b1 + b2 + + b10 , tenemos a1 + b1 + a2 + b2 + + ai + bi + ai+1 + bi+1 + + a10 + b10 = 2(a1 + a2 + + a10 ), un entero par. Colegimos que i es impar, lo que necesariamente implica a1 = b1 = 0, esto es, A y A son ambos divisibles por 10.
197 Ejemplo Demustrese que todos los enteros en la sucesin

49, 4489, 444889, 44448889, 44 . . .44 88 . . . 88 9


n4 s n1 8 s

son cuadrados. Resolucin: Obsrvese que 44 . . . 44 88 . . . 88 9


n4 s n1 8 s

= 44 . . . 44 10n + 88 . . . 88 10 + 9 = = = = 4 8 (10n 1) 10n + (10n1 1) 10 + 9 9 9 4 4 1 2n n 10 + 10 + 9 9 9 1 (2 10n + 1)2 9 2 10n + 1 2 3


n4 s n1 8 s

Falta ahora demostrar que esta ltima cantidad es entera, esto es, que 3 divide a 2 10n + 1 = 2 00 . . . 00 1. Pero la suma de los
n1 0 s

dgitos de esta ltima cantidad es 3, y por lo tanto este entero es divisible por 3.

Apendice

Indicaciones y respuestas
6 Presmase a la contraria que el conjunto S de enteros en ]0; 1[ es no nulo. Siendo un conjunto de enteros positivos, gracias al axioma del buen orden, este debe tener un elemento mnimo al que se llamar m. Ahora bien, 0 < m2 < m < 1, y por lo tanto m2 S . Pero esto declara que S tiene un entero positivo, m2 , que es menor que su elemento mnimo! Esta contradiccin establece que S = . 7 Supngase que a2 + b2 = k fuere un contraejemplo de un entero que no es un cuadrado perfecto con m x(a, b) tan pequeo a 1 + ab como fuere posible. Puede presumirse, sin perder generalidad, que a < b, ya que si a = b entonces 0<k= lo que fuerza k = 1, un cuadrado perfecto. Ahora bien, a2 + b2 k(ab + 1) = 0 es una ecuacin cuadrtica en b cuya suma de races es ka y cuyo producto de races es a2 k. Si b1 , b son sus races se tiene que b1 + b = ka y b1 b = a2 k. Como a, k son enteros positivos, el suponer b1 < 0 es incompatible con a2 + b2 = k(ab1 + 1). Como k se supone no ser 1 un cuadrado perfecto, el suponer b1 = 0 es incompatible con a2 + 02 = k(0 a + 1). Adems b1 = a2 k b2 k < < b. b b a2 + b2 1 = k y el cual es menor que m x(a, b): contradiccin. a 1 + ab1 2a2 < 2, a2 + 1

As pues, se ha encontrado otro entero positivo, b1 para el cual Por lo tanto k debe ser un cuadrado perfecto.

8 Obsrvese que n3 n = (n 1)n(n + 1) y que n5 5n3 + 4n = (n 2)(n 1)n(n + 1)(n + 2) y utilcese el ejemplo 5. 14 Dividnse los nmeros {1, 2, 3, . . . , 126} en los seis conjuntos {1, 2}, {3, 4, 5, 6}, {7, 8, . . . , 13, 14}, {15, 16, . . . , 29, 30}, {31, 32, . . . , 61, 62} y {63, 64, . . . , 126}. Por el principio de las casillas de Dirichlet, dos de los siete nmeros yacern en el mismo conjunto, donde obviamente, satisfacen las desigualdades estipuladas. 15 Hay 210 1 = 1023 subconjuntos no nulos posibles en un conjunto de 10 elementos. A cada uno de estos conjuntos no vacos asciese la suma de estos 10 elementos. El valor mximo que estas sumas pueden tener es 90 + 91 + + 99 = 945 < 1023. Luego, deben de haber al menos dos subconjuntos, llmense A y B con la misma suma, ya que hay ms subconjuntos que sumas. Si los subconjuntos tuviesen una interseccin no nula, basta considerar A \ (A B) y B \ (A B), que tambin tienen la propiedad deseada. 41

42

Captulo A

16 Obsrvese primeramente que al elegir n + 1 enteros de cualquier conjunto de 2n enteros consecutivos, siempre habr dos que diferirn por n. En efecto, al parear los 2n enteros consecutivos {a + 1, a + 2, a + 3, . . . , a + 2n} en los n pares {a + 1, a + n + 1}, {a + 2, a + n + 2}, . . ., {a + n, a + 2n}, se ve que al elegir y n + 1 siempre habr dos que pertenecen al mismo grupo. Agrupse pues los 100 enteros como sigue: {1, 2, . . . 20}, {21, 22, . . . , 40}, {41, 42, . . . , 60}, {61, 62, . . . , 80} y {81, 82, . . . , 100}. Si se eligieren 55 siempre habr once proviniendo del mismo grupo. Y en ese grupo en particular siempre habr dos diriendo por 10. 26 Se demostrar que la expresin r + 1/r es entera slo cuando r = 1, en cuyo caso r + 1/r = 2. Sea pues r+ k un entero positivo. Luego 1 = k, r

k k2 4 r= . 2

Como k es un entero, r puede ser entero si y slo si k2 4 es un cuadrado de la misma paridad que k. Ahora, si k 3, (k 1)2 < k2 4 < k2 , esto es, k2 4 est entre dos cuadrados consecutivos y por lo tanto no puede ser un cuadrado. Si k = 1, Si k = 2, k2 4 = 0. Luego, r + 1/r = 2, esto es, r = 1. Esto termina la demostracin. 27 Exprsese 11 . . . 11 de la forma
221 1 s

k2 4 no es real.

10a 1 10b 1 , para ciertos enteros a, b. 10b 1 10 1

28 1110 1 = (115 1)a = (11 1)(114 + 113 + 112 + 11 + 1)a. 29 Obsrvese que a3 b ab3 = ab(a b)(a + b) es siempre par, no importa que enteros sean substituidos. Si uno de los enteros es de la forma 5k, entonces no hay ms que demostrar. Si no, entonces se escogen tres enteros de entre enteros de la forma 5k 1 (casilla I) 5k 2 (casilla II). Por el principio de las casillas de Dirichlet, dos de estos tres enteros debern caer en la misma casilla as que hay dos que o bien su suma o bien su diferencia es divisible por 5, lo que establece el resultado. 30 Para n = 0 se declara que 33 27 == 169 0 es un mltiplo de 169, lo cual es evidente. Presmase que la asercin es cierta para n 1, n > 1, esto es, presmase que 33n 26n 1 = 169N para algn entero N. Entonces 33n+3 26n 27 = 27 33n 26n 27 = 27(33n 26n 1) + 676n lo cual se reduce a 27 169N + 169 4n, que es evidentemente un mltiplo de 169. Queda demostrada la asercin por induccin.

Indicaciones y respuestas 31 Es claro que 3n + 1 no es un mltiplo de 3, luego 3n + 1 = (3k 1)2. De aqu n+1 = como queramos demostrar. (3k 1)2 1 + 1 = 3k2 2k + 1 = k2 + k2 + (k 1)2, 3

43

32 Si n es par, escrbase n = n 6 + 6. Como n > 11, n 6 es par y mayor que 2, por tanto compuesto. Si n es impar, n = n 9 + 9. Como n > 11, n 9 es par y mayor que 2, por lo tanto compuesto. 33 Por el algoritmo de divisin, hay enteros q1 , q2 , q3 con 1059 = dq1 + r, 1417 = dq2 + r y 2312 = dq3 + r. Restando se obtiene 1253 = d(q3 q1 ), 895 = d(q3 q2 ) y 358 = d(q2 q1 ). Como 7 179, 895 = 5 179, 358 = 2 179, se ve que d = 179. Como 1059 = 5 179 + 164, r = 164. Finalmente, d r = 15. 34 Se tiene que n2 + 23 = n2 1 + 24 = (n 1)(n + 1) + 24. Luego, las familias n = 24m 1, m = 0, 1, 2, 3, . . . producen innitos valores de n2 + 23 que son divisibles por 24. 35 Se consideran enteros positivos a1 , a2 , . . . , an con a1 + a2 + + an = 1996. Es claro que para maximizar a1 a2 an , ninguna de las ak s puede ser igual a 1. Se demostrar que para obtener un producto mximo se deber tener la mayora de las ak = 3 y a lo sumo dos aj = 2. Supongse que aj > 4. Al substituir aj por los dos trminos aj 3 y 3 la suma no se afecta, pero el producto incrementa pues aj < 3(aj 3). As pues las ak s son iguales a 2, 3 4. Pero como 2 + 2 + 2 = 3 + 3 y 2 2 2 < 3 3, si hay tres o ms 2s, se pueden substituir con 3s. Como 1996 = 3(665) + 1 = 3(664) + 4, el producto mximo es pues 3664 4. 36 Gracias a 2.2, 2903n 803n es divisible por 2903 803 = 2100 = 7 300 y 261n 464n es divisible por 203 = (29) 7. Por lo tanto, la expresin es divisible por 7. Adems 2903n 464n es divisible por 2903 464 = 2439 = 9 271 y 803n + 261n es divisible por 803 + 261 = 542 = 2 271. As pues, como la expresin es divisible por 7 y por 271 y como estos son relativamente primos, la expresin es pues divisible por 7 271 = 1897. 37 Si n es impar escrbase a = 2, b = n 2, n = a + b y como n 2 es impar se tiene MCD (n 2, 2) = 1. Si n es par, entonces o bien n = 4k o bien n = 4k + 2. Si n = 4k, escrbase a = 2k + 1, b = 2k 1, n = a + b, los cuales son relativamente primos al ser dos enteros impares consecutivos. Si n = 4k + 2, k > 1 escrbase a = 2k + 3, b = 2k 1, n = a + b, los que de la misma manera son relativamente primos (si d = MCD (2k + 3, 2k 1) entonces d divide a 2k + 3 (2k 1) = 4. Luego d es 1, 2 4 pero como d es impar es por fuerza = 1). 38 Si p = 3, entonces 8p 1 = 23 y 8p + 1 = 25, luego la aseveracin se cumple para p = 3. Si p > 3, p es de la forma 3k + 1 o de la forma 3k + 2. Si p = 3k + 1 entonces 8p 1 = 24k 7 y 8p + 1 = 24k 6, que es divisible por 6 y por lo tanto no es primo. Si p = 3k + 2 entonces 8p 1 = 24k 15 no es primo al ser divisible por 3. 39 22225555 + 55552222 = (22225555 + 45555 ) + (55552222 42222 ) (45555 42222). Para otro punto de vista, vase el ejemplo 146. 40 Pngase n = pm, donde p es primo y m > 1. Utilcese la identidad 2.2 para factorizar (2p )m 1. 41 Pngase n = 2k m, donde m > 1 es impar. Factorice (22 )m + 1. 42 Si a = 103 , b = 2 entonces 1002004008016032 = a5 + a4 b + a3 b2 + a2 b3 + ab4 + b5 = Esta ltima expresin factoriza como a6 b6 ab = (a + b)(a2 + ab + b2)(a2 ab + b2 ) = 1002 1002004 998004 = 4 4 1002 250501 k, a6 b6 . ab
k

en donde k < 250000. Por lo tanto p = 250501.

44 44 Sean n 1, n, n + 1, n + 2 cuatro enteros consecutivos. Entonces su producto P es P = (n 1)n(n + 1)(n + 2) = (n3 n)(n + 2) = n4 + 2n3 n2 2n. Ahora bien, (n2 + n 1)2 = n4 + 2n3 n2 2n + 1 = P + 1 > P. Como P = 0 y P es 1 ms que un cuadrado, P no puede ser un cuadrado.

Captulo A

45 La asercin es evidente para n = 1, ya que k2 1 = (k 1)(k + 1) es producto de dos nmeros pares consecutivos, y n por tanto uno es divisible por 2 y el otro por 4 as que el producto es divisible por 8. Presmase que 2n+2 k2 1. Como k2 1 = (k2 1)(k2 + 1), se puede notar que 2n+2 divide a (k2n 1), as que el problema se reduce a demostrar que 2(k2n + 1). Pero esto es obvio, ya que como k2n es impar k2n + 1 es par. 51 Ntese que 2(21n + 4) 3(14n + 3) = 1, de donde el mximo comn divisor divide a 1. As pues, el numerador y el denominador no pueden compartir a un factor mayor que 1. 52 Se tiene que dn = MCD 100 + n2, 100 + (n + 1)2 = MCD 100 + n2, 100 + n2 + 2n + 1 = MCD 100 + n2, 2n + 1 . As pues

n+1 n n

dn (2(100 + n2) n(2n + 1)) = 200 n. Por lo tanto, dn (2(200 n) + (2n + 1)) = 401, de donde se colige que dn 401 para toda n. Alcanzar dn un valor tan grande como 401? Efectivamente! Para n = 200 se tiene que a200 = 100 + 2002 = 100(401) y que a201 = 100 + 2012 = 40501 = 101(401). Luego se deduce que m x dn = 401. a
n1

53 Pngase los 100 enteros en los 50 pares {1, 2}, {3, 4}, {5, 6} . . . , {99, 100}. Como se elegir 51 enteros, deber de haber dos de entre ellos que yazgan en el mismo par. stos son relativamente primos. 54 Sea el producto (n 1)n(n + 1) = (n2 1)n, n > 1. Como n2 1 y n son relativamente primos, por el Teorema fundamental de la aritmtica (Teorema 57) n2 1 es una potencia k-sima perfecta (k 2) y tambin lo es n. Esto implica que tanto n2 1 y n2 son potencias k-simas perfectas consecutivas, lo que es imposible. Obsrvese que ni n2 1 ni n2 son ni 0 ni 1. 63 Los nmeros de M son de la forma 2a 3b 5c 7d 11f 13g 17h 19j 23k . Los diez exponentes dan 512 = 210 vectores de paridad. Luego entre cualesquiera 513 elementos de M siempre se hallar dos cuyo producto es un cuadrado. Se poda ahora los cuadrados. Como 1985 > 513 se puede hallar un par de nmeros distintos a1 , b1 tales que a1 b1 = c2 . 1 Qutese este par, dejando 1983 enteros. De estos 1983 enteros, se puede hallar un par a2 , b2 tales que a2 b2 = c2 . Remuvase 2 este par, dejando 1981 enteros. De estos 1981 enteros, se puede encontrar un par a3 , b3 tales que a3 b3 = c2 . Esta operacin 3 de remover se puede continuar n + 1 veces, en donde n es el mayor entero positivo que satisface 1985 2n 513, esto es, n = 736. Luego se puede recoger 737 pares ak , bk tales que ak bk = ck sea un cuadrado. Como 737 > 513, se puede encontrar un par de las cm tales que su producto ci cj sea un cuadrado. Pero como cada una de las cm es un cuadrado a su vez, el producto ci cj = a2 ser una cuarta potencia.

Indicaciones y respuestas

45

64 Cualquier entero puede escribirse de la manera 2a m, en donde m es impar. Tan slo hay 50 enteros impares en el conjunto dado, luego hay slo m posibilidades para m. Luego dos de los 51 enteros elegidos deben de ser de la forma 2a m y otro de la forma 2a m. Luego el menor de estos dos dividir al mayor. 65 Ntese primeramente que 7 puede descomponerse en a lo sumo tres factores diferentes: 7 = 7(1)(1). Si p(ak ) 7 = 0 para cuatro valores distintos ak , 1 k 4 entonces p(x) 7 = (x a1 )(x a2 )(x a3 )(x a4 )q(x) para algn polinomio q con coecientes enteros. Presmase ahora que existe un entero m con p(m) = 14. Entonces 7 = p(m) 7 = (m a1 )(m a2 )(m a3 )(m a4 )q(m). Como los factores m ak , la igualdad anterior descompone a 7 en cuatro o ms valores distintos, lo que es una contradiccin. 66 Obsrvese que m5 + 3m4 n 5m3 n2 15m2 n3 + 4mn4 + 12n5 = (m 2n)(m n)(m + n)(m + 2n)(m + 3n), un producto de cinco factores. Ahora bien, 33 se puede descomponer a lo sumo en cuatro factores distintos: 33 = (11)(3)(1)(1). Si n = 0, todos los factores de arriba son distintos y no pueden dar 33 en virtud del Teorema fundamental de la aritmtica (Teorema 57) porque un producto de cinco factores diferentes no puede igualar a un producto de cuatro factores diferentes. Si n = 0, el producto de los factores es m5 pero 33 no es una quinta potencia. 67 Sea k el mximo entero que satisface 2k n y sea P el producto de todos los enteros impares menores o iguales a n. El 1 nmero 2k1 PS es una suma cuyos trminos, excepto el 2k1 PS k , son enteros. 2 68 Si k2 = 28 + 211 + 2n = 2304 + 2n = 482 + 2n , entonces k2 482 = (k 48)(k + 48) = 2n . Gracias a la propiedad de factorizacin nica, k 48 = 2s , k + 48 = 2t , s + t = n. Luego 2t 2s = 96 = 3 25 o 2s (2ts 1) = 3 25 . Por factorizacin nica, s = 5, t s = 2, dando s + t = n = 12. 92 400 151 Quirese hallar 3100 m d 10. Obsrvese que 32 1 m d 10. Luego, 3100 = (32 )50 (1)50 1 m d 10. As, o o o el ltimo dgito es el 1. 152 Se tiene que hallar 77 m d 10. Ahora bien, como 72 1 m d 10, entonces tenemos 73 72 7 7 3 m d 10 o o o y 74 (72 )2 1 m d 10. Adems, 72 1 m d 4 y por lo tanto 77 (72 )3 7 3 m d 4, lo que quiere decir que hay o o o un entero t tal que 77 = 3 + 4t. Ensamblando todo esto, 77 74t+3 (74 )t 73 1t 3 3 As el ltimo dgito es un 3. 153 Obsrvese que 32n+1 3 9n 3 2n m d 7 y 2n+2 4 2n m d 7. Luego o o 32n+1 + 2n+2 7 2n 0 para todo nmero natural n. 154 Como 30a0b03 = 3 + 100b + 10000a + 3000000, observamos que 30a0b03 3 + 9b + 3a + 3 6 + 9b + 3a mod 13. Para que 30a0b03 sea divisible por 13 necesitamos 9b + 3a 7 mod 13. Aqu claro est, se tendr 0 a, b 9. Por inspeccin se ve que a = 8, b = 1;a = 5, b = 2; a = 2, b = 3; a = 9, b = 5; a = 6, b = 6; a = 3, b = 7; a = 0, b = 8. Luego 3080103, 3050203, 3020303, 3090503, 3060603, 3030703, 3000803 son todos divisibles por 13. 155 Si x2 = 2 5y2 , entonces x2 2 m d 5. Pero 2 no es un cuadrado m d 5. o o m d 7, o
7 7

m d 10. o

46

Captulo A

156 Obsrvese que 641 = 27 5 + 1 = 24 + 54 . Luego 27 5 1 m d 641 y 54 24 m d 641. Ahora bien, 27 5 o o 1 m d 641 nos da 54 228 = (5 27 )4 (1)4 1 m d 641. Esta tima congruencia y 54 24 mod 641 nos da o o 24 228 1 m d 641, lo que signica que 641(232 + 1). o

157 Obsrvese que 21 2, 22 4, 23 1, 24 2, 25 4, 26 1 m d 7 y as 23k 1 m d 3 para todos los enteros o o positivos k. Luego 23k + 27 1 + 27 0 m d 7 para todos los enteros positivos k. Esto produce una familia innita de o enteros n = 3k, k = 1, 2, . . . tal que 2n + 27 es divisible por 7. 158 No. Los cubos m d 7 son 0, 1, y 6. Ahora bien, cada potencia de de 2 es congruente con 1, 2, 4 m d 7. As pues, o o 2y + 15 2, 3, 5 m d 7. Esto es imposible. o 159 21 2, 22 4, 23 1 m d 7 y este ciclo de tres se repite. As pues, 2k 5 deja residuos 3, 4, 6 al ser dividido por 7. o

Anda mungkin juga menyukai